Концепция «релятивистской массы» существует почти столько же, сколько и теория относительности. Но является ли она разумным способом осмысления вещей?

Независимо от того, кто вы, где вы находитесь и как быстро вы движетесь, законы физики будут выглядеть для вас точно так же, как и для любого другого наблюдателя во Вселенной. Эта концепция, согласно которой законы физики не меняются при перемещении из одного места в другое или из одного момента в другой, известна как принцип относительности и восходит не к Эйнштейну, а ещё дальше в прошлое: по крайней мере, ко временам Галилея. Если на объект действует сила, то он ускоряется (т.е. изменяет свой импульс), причём величина ускорения напрямую зависит от силы, действующей на объект, делённой на его массу. На языке математики это утверждение выглядит как знаменитое уравнение Ньютона F = ma: сила равна массе, умноженной на ускорение.

Но когда были открыты частицы, движущиеся со скоростью, близкой к скорости света, неожиданно возникло противоречие. Если на малую массу действует слишком большая сила, а силы вызывают ускорение, то массивный объект можно разогнать до скорости света или даже превысить её! Это, конечно, невозможно, и именно теория относительности Эйнштейна позволила нам разрешить этот парадокс. Обычно это объясняется через понятие так называемой «релятивистской массы», т.е. тем, что по мере приближения к скорости света масса объекта увеличивается, поэтому та же сила вызывает меньшее ускорение, не позволяя достичь скорости света. Но верна ли такая интерпретация «релятивистской массы»? Только отчасти. Вот научный ответ на этот вопрос.

В вакууме космоса весь свет, независимо от длины волны и энергии, распространяется с одной и той же скоростью — скоростью света в вакууме. Когда мы наблюдаем свет от далёкой звезды, мы наблюдаем свет, который уже прошёл путь от источника до наблюдателя.
В вакууме космоса весь свет, независимо от длины волны и энергии, распространяется с одной и той же скоростью — скоростью света в вакууме. Когда мы наблюдаем свет от далёкой звезды, мы наблюдаем свет, который уже прошёл путь от источника до наблюдателя.

Первое, что важно понять, — это то, что принцип относительности, независимо от того, как быстро вы движетесь или где находитесь, остаётся верным всегда: законы физики действительно одинаковы для всех, независимо от того, где вы находитесь и когда проводите измерения. Эйнштейн знал (чего не могли знать ни Ньютон, ни Галилей) следующее: скорость света в вакууме должна быть абсолютно одинаковой для всех. Это потрясающее осознание, которое противоречит нашим интуитивным представлениям о мире.

Представьте, что у вас есть автомобиль, который может двигаться со скоростью 100 км/ч. Представьте, что к этому автомобилю прикреплена пушка, которая может разогнать пушечное ядро из состояния покоя до точно такой же скорости: 100 километров в час. Теперь представьте, что ваш автомобиль движется и вы стреляете из пушки, при этом вы можете контролировать, в какую сторону направлена пушка.

  • Если направить пушку в ту же сторону, куда движется автомобиль, то ядро будет двигаться со скоростью 200 км/ч: скорость автомобиля плюс скорость пушечного ядра.

  • Если направить пушку вперёд и вверх, в то время как автомобиль движется вперёд, то ядро будет двигаться со скоростью 141 км/ч: комбинация движения вперёд и вверх под углом 45°.

  • А если направить пушку в обратном направлении, выстрелив ядром назад, в то время как автомобиль движется вперёд, то ядро будет двигаться со скоростью 0 км/ч: две скорости в точности аннулируют друг друга.

Как продемонстрировали в одном из эпизодов программы «Разрушители мифов», снаряд, выпущенный назад из движущегося вперёд автомобиля с точно такой же скоростью, будет выглядеть падающим прямо вниз в состоянии покоя; скорость грузовика и скорость вылета из «пушки» в этом случае в точности аннулируют друг друга.
Как продемонстрировали в одном из эпизодов программы «Разрушители мифов», снаряд, выпущенный назад из движущегося вперёд автомобиля с точно такой же скоростью, будет выглядеть падающим прямо вниз в состоянии покоя; скорость грузовика и скорость вылета из «пушки» в этом случае в точности аннулируют друг друга.

Это то, что мы обычно наблюдаем, и это соответствует нашим ожиданиям. И это также экспериментально верно, по крайней мере, для нерелятивистского мира. Но если бы мы заменили пушку фонариком, история была бы совсем другой. Можно взять автомобиль, поезд, самолёт или ракету, движущуюся с любой скоростью, и посветить из неё фонариком в любом направлении.

Фонарик будет излучать фотоны со скоростью света, или 299 792 458 м/с, и эти фотоны всегда будут двигаться с той же самой скоростью (в вакууме).

  • Вы можете пустить фотоны в том же направлении, в котором движется ваш автомобиль, и они всё равно будут двигаться со скоростью 299 792 458 м/с.

  • Можно выстрелить фотонами под углом к направлению движения, и хотя это может изменить направление движения фотонов, они всё равно будут двигаться с той же скоростью: 299 792 458 м/с.

  • А можно выстрелить фотонами прямо противоположно направлению движения, и всё равно они будут двигаться со скоростью 299 792 458 м/с.

Скорость, с которой движутся фотоны, будет такой же, как и всегда, скоростью света, причём не только с вашей точки зрения, но и с точки зрения любого наблюдателя. Единственное различие, которое кто-то увидит в зависимости от того, с какой скоростью движутся вы (излучатель) и он (наблюдатель), — это длина волны света: более красная (большая длина волны), если вы взаимно удаляетесь друг от друга, и более голубая (меньшая длина волны), если вы взаимно приближаетесь друг к другу.

У объекта, движущегося со скоростью, близкой к скорости света и излучающего свет, излучаемый им свет будет выглядеть смещённым в зависимости от местоположения наблюдателя. Тот, кто находится слева, увидит, что источник удаляется от него, и, следовательно, свет будет красным; тот, кто находится справа от источника, увидит, что он синеет, или смещается к более высоким частотам, поскольку источник движется к нему.
У объекта, движущегося со скоростью, близкой к скорости света и излучающего свет, излучаемый им свет будет выглядеть смещённым в зависимости от местоположения наблюдателя. Тот, кто находится слева, увидит, что источник удаляется от него, и, следовательно, свет будет красным; тот, кто находится справа от источника, увидит, что он синеет, или смещается к более высоким частотам, поскольку источник движется к нему.

Это ключевое осознание пришло в голову Эйнштейну, когда он разрабатывал свою первоначальную специальную теорию относительности. Он попытался представить себе, как будет выглядеть свет, который, как он знал, является электромагнитной волной, для человека, движущегося за этой волной со скоростью, близкой к скорости света.

Хотя мы нечасто думаем об этом в таких терминах, тот факт, что свет является электромагнитной волной, означает:

  • что эта световая волна несёт в себе энергию,

  • что при распространении в пространстве она создаёт электрические и магнитные поля,

  • эти поля колеблются, синфазно и под углом 90° друг к другу,

  • и когда они проходят мимо других заряженных частиц, например электронов, они могут заставить их периодически двигаться, поскольку заряженные частицы испытывают действие сил (и, следовательно, ускорения), когда на них действуют электрические и/или магнитные поля.

Это было установлено в 1860-1870-х годах, в результате работ Джеймса Клерка Максвелла, уравнений которого до сих пор достаточно для описания классического электромагнетизма. Вы ежедневно пользуетесь этой технологией: каждый раз, когда антенна «улавливает» сигнал, он возникает благодаря заряженным частицам в антенне, движущимся в ответ на электромагнитные волны.

Свет — это не что иное, как электромагнитная волна с синфазно колеблющимися электрическим и магнитным полями, перпендикулярными направлению распространения света. Чем короче длина волны, тем энергичнее фотон, но тем больше он подвержен изменениям скорости распространения света в среде.
Свет — это не что иное, как электромагнитная волна с синфазно колеблющимися электрическим и магнитным полями, перпендикулярными направлению распространения света. Чем короче длина волны, тем энергичнее фотон, но тем больше он подвержен изменениям скорости распространения света в среде.

И независимо от того, кто вы, где вы, когда вы и как быстро вы движетесь, вы и все остальные всегда видите, что свет движется с одной и той же скоростью: скоростью света.

Но не все свойства света одинаковы для всех наблюдателей. Тот факт, что наблюдаемая длина волны света меняется в зависимости от того, как движутся относительно друг друга источник и наблюдатель, означает, что должны меняться и некоторые другие характеристики света.

  • Частота света должна меняться, поскольку частота, умноженная на длину волны, всегда равна скорости света, которая является постоянной величиной.

  • Энергия каждого кванта света должна меняться, поскольку энергия каждого фотона равна постоянной Планка (которая является константой), умноженной на частоту.

  • И импульс каждого кванта света также должен меняться, поскольку импульс (для света) равен энергии, делённой на скорость света.

Шкалы размеров, длин волн и температур/энергий, соответствующие различным частям электромагнитного спектра. Для исследования мельчайших масштабов необходимо переходить к более высоким энергиям и более коротким длинам волн. Ультрафиолетовый свет достаточен для ионизации атомов, но по мере расширения Вселенной свет систематически смещается к более низким температурам и большим длинам волн.
Шкалы размеров, длин волн и температур/энергий, соответствующие различным частям электромагнитного спектра. Для исследования мельчайших масштабов необходимо переходить к более высоким энергиям и более коротким длинам волн. Ультрафиолетовый свет достаточен для ионизации атомов, но по мере расширения Вселенной свет систематически смещается к более низким температурам и большим длинам волн.

У света, напомним, может быть огромный энергетический диапазон: от гамма-лучей с самыми высокими энергиями до рентгеновских лучей, ультрафиолетового света, видимого света (от фиолетового до синего, зелёного, жёлтого, оранжевого и красного), инфракрасного света, микроволнового света и, наконец, радиоизлучения с самыми низкими энергиями. Чем выше энергия на один фотон, тем короче длина волны, выше частота и больше импульс; чем ниже энергия на один фотон, тем больше длина волны, ниже частота и меньше импульс.

Свет также может, как показал сам Эйнштейн в 1905 г., исследуя фотоэлектрический эффект, передавать энергию и импульс материи, то есть массивным частицам. Если бы единственным законом был закон Ньютона в том виде, в котором мы привыкли его видеть, когда сила равна массе, умноженной на ускорение (F = ma), свет оказался бы в затруднительном положении. При отсутствии массы, присущей фотонам, это уравнение не имело бы никакого смысла. Но сам Ньютон писал не уравнение «F = ma», как мы часто полагаем, а утверждение, что «сила — это скорость изменения импульса во времени», или что приложение силы вызывает «изменение импульса» во времени.

Внутри БАК протоны пролетают друг мимо друга со скоростью 299 792 455 м/с, что всего на 3 м/с меньше скорости света. Ускорители частиц, подобные БАКу, состоят из секций ускоряющих полостей, в которых действуют электрические поля, ускоряющие находящиеся в них частицы, и кольцевых изгибающихся секций, в которых действуют магнитные поля, направляющие быстро движущиеся частицы либо к следующей ускоряющей полости, либо к точке столкновения.
Внутри БАК протоны пролетают друг мимо друга со скоростью 299 792 455 м/с, что всего на 3 м/с меньше скорости света. Ускорители частиц, подобные БАКу, состоят из секций ускоряющих полостей, в которых действуют электрические поля, ускоряющие находящиеся в них частицы, и кольцевых изгибающихся секций, в которых действуют магнитные поля, направляющие быстро движущиеся частицы либо к следующей ускоряющей полости, либо к точке столкновения.

Так что же это значит — импульс? Хотя у многих физиков есть своё собственное определение, мне всегда нравилось следующее: «Это мера количества вашего движения». Если представить себе причал, то можно вообразить, как будут выглядеть столкновения с этим причалом различных объектов.

  • Шлюпка может двигаться как относительно медленно, так и быстро, но благодаря малой массе её импульс будет оставаться небольшим. Сила, которую она оказывает на док при столкновении, будет ограничена, и только самые слабые доки получат какие-либо структурные повреждения при столкновении с мелкой лодкой.

  • В ситуации со стрельбой из огнестрельного оружия в доке всё будет по-другому. Несмотря на то, что снаряды — пули, пушечные ядра или что-то более разрушительное, например артиллерийские снаряды — могут иметь небольшую массу, они будут двигаться с очень большой (но всё же нерелятивистской) скоростью. При массе, равной 0,01%, но скорости, равной 10000% скорости лодки, их импульсы могут быть схожими, но сила будет воздействовать на гораздо меньшую площадь. Структурные повреждения будут значительными, но только в отдельных местах.

  • Или можно столкнуть с причалом на чрезвычайно малой скорости медленно движущийся, но массивный объект, например круизный лайнер, суперъяхту или линкор. При массе, в миллионы раз превышающей массу шлюпки — они могут весить десятки тысяч тонн — даже ничтожная скорость может привести к полному разрушению дока. Импульс объектов с большой массой — это не шуточки.

Крупная суперъяхта MotorYacht GO врезалась в док яхт-клуба в Сен-Мартене. Из-за большого импульса яхта, разрушив док, пробила дерево, бетон и даже армированную сталь. Для очень больших масс, движущихся даже с небольшой скоростью, импульс может быть катастрофическим.
Крупная суперъяхта MotorYacht GO врезалась в док яхт-клуба в Сен-Мартене. Из-за большого импульса яхта, разрушив док, пробила дерево, бетон и даже армированную сталь. Для очень больших масс, движущихся даже с небольшой скоростью, импульс может быть катастрофическим.

Проблема заключается в том, что, как было известно ещё Ньютону, сила, действующая на что-то, равна изменению импульса во времени. Если на объект действует сила в течение определённого времени, то это приведёт к изменению его импульса на определённую величину. Это изменение зависит не от скорости движения объекта, а только от «количества движения», которым он обладает, — его импульса.

Так что же происходит с импульсом объекта, когда он приближается к скорости света? Именно это мы и пытаемся понять, когда говорим о силе, импульсе, ускорении и скорости при приближении к скорости света. Если объект движется с 50% скорости света и у него есть пушка, способная выстреливать снаряд с 50% скорости света, что произойдёт, когда обе скорости будут направлены в одну сторону?

Вы знаете, что скорость света для массивного объекта недостижима, поэтому наивная мысль о том, что «50% скорости света + 50% скорости света = 100% скорости света», должна быть ошибочной. Но сила, действующая на пушечное ядро, при выстреле из релятивистски движущейся системы отсчёта изменит его импульс ровно на ту же величину, что и при выстреле из состояния покоя. Если выстрел пушечным ядром, находившимся в состояния покоя, изменяет его импульс на определённую величину, в результате чего оно приобретает скорость, равную 50% скорости света, то выстрел по нему из положения, когда оно уже движется со скоростью 50% скорости света, должен изменить его импульс на ту же величину. Почему же тогда его скорость не будет равна 100% скорости света?

Моделирование релятивистского путешествия к созвездию Ориона на различных скоростях. По мере приближения к скорости света пространство не только искажается, но и сокращается расстояние до звёзд, а время в пути становится меньше. Для создания иллюстраций «Ориона» была использована программа StarStrider — релятивистский 3D-планетарий компании FMJ-Software. Для того чтобы преодолеть 1000 с лишним световых лет менее чем за 1000 лет, необязательно нарушать скорость света — но это только с вашей точки зрения.
Моделирование релятивистского путешествия к созвездию Ориона на различных скоростях. По мере приближения к скорости света пространство не только искажается, но и сокращается расстояние до звёзд, а время в пути становится меньше. Для создания иллюстраций «Ориона» была использована программа StarStrider — релятивистский 3D-планетарий компании FMJ-Software. Для того чтобы преодолеть 1000 с лишним световых лет менее чем за 1000 лет, необязательно нарушать скорость света — но это только с вашей точки зрения.

Понимание ответа является ключом к пониманию теории относительности: всё дело в том, что «классическая» формула для импульса — импульс равен массе, умноженной на скорость — является лишь нерелятивистским приближением. В реальности необходимо использовать формулу для релятивистского импульса, которая несколько отличается и включает в себя коэффициент, который физики обозначают (γ) — фактор Лоренца, который увеличивается тем больше, чем ближе вы к скорости света. Для быстро движущейся частицы импульс — это не просто масса, умноженная на скорость, а масса, умноженная на скорость, умноженная на гамму.

При приложении той же силы, что и к покоящемуся объекту, к движущемуся объекту, даже релятивистскому, его импульс изменится на ту же величину, но весь этот импульс не пойдёт на увеличение его скорости, а часть его пойдёт на увеличение гаммы — коэффициента Лоренца. В предыдущем примере ракета, движущаяся со скоростью 50% скорости света и выстреливающая пушечное ядро со скоростью 50% скорости света, приведёт к тому, что пушечное ядро будет лететь со скоростью 80% скорости света, а коэффициент Лоренца составит 1,6667. Идея «релятивистской массы» очень стара и была популяризирована Артуром Эддингтоном, астрономом, чья экспедиция в 1919 г. для наблюдения солнечного затмения подтвердила теорию общей относительности Эйнштейна, но она допускает определённую вольность: в ней предполагается, что коэффициент Лоренца (γ) и масса покоя (m) увеличиваются вместе, и это предположение не может быть проверено никакими физическими измерениями или наблюдениями.

Замедление времени (слева) и сокращение длины (справа) показывают, что время идёт медленнее, а расстояния становятся тем меньше, чем ближе мы к скорости света. По мере приближения к скорости света часы всё сильнее замедляются, а расстояния сокращаются до бесконечно малых величин.
Замедление времени (слева) и сокращение длины (справа) показывают, что время идёт медленнее, а расстояния становятся тем меньше, чем ближе мы к скорости света. По мере приближения к скорости света часы всё сильнее замедляются, а расстояния сокращаются до бесконечно малых величин.

Смысл всего этого в том, чтобы понять, что при движении со скоростью, близкой к скорости света, существует множество важных величин, которые больше не подчиняются нашим классическим уравнениям. Вы не можете просто сложить скорости, как это делали Галилей или Ньютон; вы должны сложить их релятивистски.

Нельзя считать расстояния фиксированными и абсолютными; необходимо понимать, что они сокращаются вдоль направления движения. И даже нельзя считать, что время для вас проходит так же, как и для того, кто за вами наблюдает — оно относительно и замедляется для наблюдателей, движущихся с разными относительными скоростями.

Световые часы, образованные фотоном, отражающимся от двух зеркал, определяют время для любого наблюдателя. Хотя два наблюдателя могут не соглашаться друг с другом в том, сколько времени проходит, они будут согласны с законами физики и константами Вселенной, такими как скорость света. При правильном применении теории относительности окажется, что их измерения эквивалентны друг другу. Явление замедления времени, впервые выведенное Лоренцем в 1890-х годах, вскоре привело Эйнштейна к открытию специальной теории относительности.
Световые часы, образованные фотоном, отражающимся от двух зеркал, определяют время для любого наблюдателя. Хотя два наблюдателя могут не соглашаться друг с другом в том, сколько времени проходит, они будут согласны с законами физики и константами Вселенной, такими как скорость света. При правильном применении теории относительности окажется, что их измерения эквивалентны друг другу. Явление замедления времени, впервые выведенное Лоренцем в 1890-х годах, вскоре привело Эйнштейна к открытию специальной теории относительности.

Заманчиво, но в конечном счёте неверно сваливать несоответствие между классическим и релятивистским миром на идею релятивистского изменения массы. Для массивных частиц, движущихся со скоростью, близкой к скорости света, эту концепцию ещё можно применить, чтобы понять, почему объекты могут приближаться к скорости света, но не достигать её, но она рассыпается, как только вы включаете в рассмотрение безмассовые частицы, такие как фотоны.

Гораздо лучше понимать законы относительности такими, какие они есть на самом деле, чем пытаться втиснуть их в более интуитивные рамки, применение которых принципиально ограничено. Как и в случае с квантовой физикой, пока вы не проведёте достаточно времени в мире относительности, чтобы получить интуитивное представление о том, как всё работает, слишком упрощённая аналогия поможет вам только в одном случае. Когда вы достигнете её пределов, вы пожалеете, что не изучили её правильно и всесторонне с первого раза.

Комментарии (173)


  1. lgorSL
    30.10.2023 17:26
    +20

    В реальности необходимо использовать формулу для релятивистского импульса

    Не вижу такой формулы в статье. Вижу только F=ma из школьной программы седьмого класса. Для кого этот текст написан?

    Ну есть же в теоретической физике красивая стройная математика - преобразования Лоренца, пространство Минковского. Есть интересные исторические эксперименты, например ещё в 1676 Рёмер понаблюдал за спутниками Юпитера и смог довольно точно оценить скорость света.
    Почему бы не написать про это?


    1. Pshir
      30.10.2023 17:26
      +5

      В вашем вопросе и содержится ответ. Эта статья написана для тех, кто ещё не добрался до 11 класса школы, где проходится всё то, что написано здесь.


      1. pavelsc
        30.10.2023 17:26
        +2

        Я ещё в том самом 11 классе почувствовал себя тупым, когда весь класс внезапно понял СТО и ОТО и почему масса растет, длина уменьшается. И не поднял руку и не попросил объяснить. И на протяжении многих лет внимал тем, кто ловко объяснял теорию относительности в компаниях, так и не отваживаясь спросить подробнее, боясь показаться невеждой.

        А потом уже, через 15 лет после школы, начал интересоваться этим, читая разнообразные записи и слушая аудиокниги на тему по несколько часов в день. И наткнулся на один интересный исторический дискурс:

        Знаменитого английского астрофизика Артура Эддингтона спросили:
        – Сэр, правду ли говорят, что вы один из трёх человек в мире, которые понимают теорию относительности Эйнштейна?

        Наступило долгое и неловкое молчание – учёный явно затруднялся с ответом. Тогда спрашивающий поспешил исправить положение: – Может быть, сэр, я что-то не так сказал? Мне, видимо, следовало бы догадаться, что вы, сэр, при всей вашей скромности, сочтёте мой вопрос несколько бестактным. В таком случае, сэр, позвольте принести вам мои самые искренние извинения…

        – Ничего-ничего, – благодушно ответил Эддингтон, – Просто я задумался, пытаясь вспомнить, кто же этот третий…

        ------------

        Моё почтение, Pshir, судя по всему этим третьим являетесь вы ))


        1. Pshir
          30.10.2023 17:26

          Я не говорю о том, что я идеально всё понимаю. Тем более, что здесь нет никакой ОТО. Здесь рассказываются совершенно элементарные вещи, которые и тогда понимала куча людей, а не три. Я говорю о том, что в школьном учебнике физики данный предмет освещается лучше, чем в этой статье.


        1. ksbes
          30.10.2023 17:26
          +1

          В СТО - на самом деле там всё проще чем кажется. Надо просто понять, что у летящего стержня передний конец из прошлого, а задний из будущего. От этого и плясать.

          А вот с ОТО - там надо понять, что когда говорят обо всех этих метриках и искажениях пространства - то речь идёт об искажении 4D псевдоеклидова пространства (тут, к сожалению надо знать что такое псевдовектор). Причём абсолютно статичного (нет "второго времени" в котором оно может менятся). А вовсе не "обычного 3D" .
          Затем понять что 4D это слишком сложно, чтобы это всё понимать, если только не профессионально этим занимаешься и забить.


    1. u007
      30.10.2023 17:26
      +1

      Продолжение для любопытных тут: Почему E = mc² — это лишь половина происходящего


  1. diakin
    30.10.2023 17:26
    -7

    Просто само наше представление о пространстве.. оно не то чтобы неверно..
    Ну вот допустим есть звуковые волны, они распространяются в среде.. допустим в металлическом цилиндре который летает в космосе. С "точки зрения" этих волн металлический цилиндр является пустым пространством, а вакуум вне его для них является твердью, непреодолимой преградой, в которую они проникнуть не могут. Возможно и наше "пустое пространство" с какой-нибудь точки зрения является чем-то иным.


    1. zbot
      30.10.2023 17:26
      -1

      имеешь ввиду что как со сверхзвуком - газ в пол и в какой-то момент хлоп... бах и на спидометре уже 350 м/с?


      1. diakin
        30.10.2023 17:26
        -2

        Бах? Дануна..


      1. VPryadchenko
        30.10.2023 17:26
        +3

        Не бах, а мах :)


        1. diakin
          30.10.2023 17:26
          -1

          "Обнаженная махом" (с)


          1. diakin
            30.10.2023 17:26

            Для тех кто не в курсе мема. Был такой фильм "Обнаженная Маха" 1958г. Показывали в том числе в СССР. А в сельский клуб название фильмов сообщали по телефону, чтобы афишу заранее подготовить. Кто-то что-то не так услышал, и на афише появилась "Обнаженная махом" (см. журнал "Крокодил" за 196х год)
            Оттуда же "Потомок Умульду" и др.


      1. Daemon23RUS
        30.10.2023 17:26
        +2

        Напомнило известную историю в Аризоне.

        Из отчета дорожного патруля штата Аризона

        Похоже, бывшему сержанту ВВС удалось где-то достать реактивный ускоритель взлета. Сержант отправился в Аризонскую пустыню, прихватив ракетный ускоритель, и выбрал длинный прямой участок дороги. Приделал ускоритель к своему автомобилю, запрыгнул в него, разогнался до приличной скорости и запустил ракету.

            Фактические данные, какие удалось восстановить, следующие. Водитель транспортного средства управлял автомобилем "шевроле импала" 1967 года выпуска. Он запустил реактивный ускоритель взлета примерно в 6,3 км от места аварии. Это удалось установить по отчетливой полосе обожженного и расплавленного асфальта. автомобиль быстро развил скорость 400-480 км/ч и продолжал двигаться с такой скоростью, на полной мощности, еще 20-25 секунд. "Без пяти минут пилот" испытывал перегрузки, подобные тем, что возникают на борту F-14 во время воздушного боя на полном форсаже. "Шевроле" продержался на прямом участке шоссе примерно 4,2 км (от 15 до 20 секунд) до того, как водитель начал тормозить, полностью расплавив тормоза и содрав шины; на асфальте остались заметные следы резины.
            Затем автомобиль оторвался от земли; он пролетел по воздуху еще 2 километра, столкнулся со скалой на высоте 38 метров и оставил в камне почерневшую воронку глубиной 90 см.


        1. diakin
          30.10.2023 17:26

          Водитель успел катапультироваться?


    1. V_Scalar
      30.10.2023 17:26
      -2

      Без инерциальных систем отсчёта в этих вопросах не разобраться, для этого надо знать как работает механизм инерции. Механизм инерции это — механизм динамической генерации масс Хиггса.

      Вселенная находится в «фазе Хиггса», и эта фраза опирается на очень элегантную связь между квантовой теорией поля частиц и статистической теорией поля систем конденсированного состояния. Точно так же, как мы можем обсуждать фазовые переходы между жидким и газообразным состояниями (или более сложными фазами), мы также можем обсуждать, как Вселенная претерпела электрослабый фазовый переход, который привел к хиггсовской скорости, которая придает массу нашим любимым частицам

      смотри ка, уже вырисовывается что то вразумительное, был непостижимый уму фотон с двумя поперечными поляризациями, потом он съел голдстоуноский бозон и у него появилась третья продольная степень поляризации, теперь частица подчиняется молекулярно кинетической теории, сталкивается, отскакивает


  1. MishaRash
    30.10.2023 17:26
    +4

    Аргументация кажется слабой. Я бы в первую очередь подчеркнул, что хоть для изменяющейся со скоростью движения центра масс релятивистской массой продолжает выполняться p=mv и работает известная (вероятно, даже слишком) формула E=mc^2,

    1. F=ma (более конкретно, dp/dt = m dv/dt) с ней не работает, как и с другими скалярными величинами, потому что сила и ускорение в общем релятивистком случае вообще не параллельны.

    2. Использовать эту массу как источник гравитационного поля в ньютоновских формулах тоже некорректно, так как на релятивистских скоростях импульс и давление тоже значительны и вносят свой вклад в гравитационное поле согласно уравнениям Эйнштейна в ОТО.

    Так что получается практически лишняя величина, сводящаяся к энергии.


    1. Tyusha
      30.10.2023 17:26
      +2

      По поводу 1. Не поняла. Что вы имеете в виду, когда говорите о не параллельности? Каково тогда по-вашему определение силы в релятивистской случае, если она не параллельна ускорению?

      Второй закон Ньютона можно заменить уравнением Лагранжа, где сила это ∂L/∂q? У меня других предложений нет.

      Или вы имеете в виду запаздывающие потенциалы?


      1. MishaRash
        30.10.2023 17:26
        +3

        F=ma работает, но с 4-силой, массой покоя (тела или системы тел) и 4-ускорением. В 4-силе и 4-ускорении фигурирует собственное время. Люди, не знакомые с теорией относительности, скорее подумают про 3-силу и 3-ускорение, обычные векторы с простым координатным временем, для которых всё сложнее — возникают разные "продольная" и "поперечная" массы (последняя всё же совпадает с "релятивистской" массой достаточно естественным образом, но всё равно в общем случае векторы не сонаправлены, только если оба строго параллельны или перпендикулярны скорости). В английской Википедии нужные формулы есть.


  1. Tzimie
    30.10.2023 17:26
    +13

    На самом деле забыто главное. Понятие "релятивистской массы" давно не используется физиками. Это такой артефакт из начала 20 века


    1. RichardMerlock
      30.10.2023 17:26
      +2

      Более того, сам Ньютон понимал массу именно как количество вещества. Так что имеет смысл на этой пропорциональной зависимости и оставаться во избежание разночтений.


  1. SergioPrieto
    30.10.2023 17:26

    Превью статьи из Elite великолепно!!!!


  1. ababo
    30.10.2023 17:26
    +1

    да


  1. devspec
    30.10.2023 17:26

    И всё-таки не совсем ясно (наверное, не только мне?).

    Взяли мы два космических корабля. Один стартанул в одну сторону, другой в другую. Оба достигли скорости 50% световой. Когда второй корабль достигнет 51% скорости света - это ведь будет означать, что он движется относительно первого со скоростью 101% скорости света?

    Оба корабля имеют конечную массу. Теоретически разогнать до 51% скорости света корабль возможно.

    Так что же будет?


    1. VBDUnit
      30.10.2023 17:26
      +13

      241 267 237 м/с. Суммарная скорость в ТО никогда не превышает скорости света:

      Релятивистское сложение скоростей
      Релятивистское сложение скоростей

      Вообще в ТО важно уточнять для какого наблюдателя мы обсчитываем явление. Потому что разные наблюдатели будут видеть разную картинку, в том числе, одновременные для одного наблюдателя события могут не быть одновременными для другого.


      1. devspec
        30.10.2023 17:26
        +1

        Спасибо за ответ


      1. MishaRash
        30.10.2023 17:26
        +9

        Справедливости ради нужно отметить, что с точки зрения "неподвижного" наблюдателя в точке старта космических кораблей расстояние между ними будет увеличиваться с просто суммарной скоростью, которая может быть больше скорости света. Но это не помешает кораблям общаться, т.к. (опять же, с точки зрения того "неподвижного" наблюдателя) один из них испустит сигнал в фиксированной точке и тому свету нужно будет лишь догнать второй, который удаляется медленнее скорости света. А с точки зрения одного корабля он неподвижен, а другой движется тоже медленнее скорости света.


    1. kenoma
      30.10.2023 17:26

      Сложение скоростей будет, но не по привычной формуле, а по Лоренцу, в учет идет еще и время на корабле, которое замедляется с ростом скорости.


    1. ilriv
      30.10.2023 17:26
      +4

      Ничего не будет.

      Если первый корабль будет иметь скорость 50% от световой, а второй 51% - их скорость относительно друг друга всё равно будет меньше скорости света. Потому что их скорости складываются не по простой формуле v1+v2.

      Причину по которой простое суммирование не работает, проще всего уловить из геометрических соображений. Дело в том, что перемещения происходят на самом деле не в 3-мерном пространстве x+y+z, а в 4-мерном пространстве x+y+z+c*t. С течением времени мы смещаемся по оси времени относительно начальной точки. Это установлено экспериментально. На малых скоростях этот нюанс неважен, но на больших скоростях мы должны делать все расчеты в 4-мерной геометрии. А там формула v1+v2 уже не сработает т.к. она не учитывает перемещение по оси времени. Все законы Специальной теории относительности Эйншейна - это результат 4-мерности пространства.


      1. StjarnornasFred
        30.10.2023 17:26
        -8

        Дело в том, что перемещения происходят на самом деле не в 3-мерном пространстве x+y+z, а в 4-мерном пространстве x+y+z+c*t

        Вот это уже лишнее. Мало того что сама идея времени как четвёртого измерения в принципе является лишь фигурой речи и художественным описанием, а не физическим явлением, так ещё и приведённая формула, скажем так, неточна и лишь отвлекает. (Не говоря уже о том, что даже в этой концепции 4-мерная геометрия вообще ни при чём)

        Чуть выше уже привели формулу релятивистского сложения скоростей - ею и надо пользоваться, не вводя лишних сущностей и мыслительных конструкций.


        1. Hardcoin
          30.10.2023 17:26
          +5

          Бездумно пользоваться формулами достаточно глупо, если вы хотите понять происходящее. Формулы не из священной книги берутся, их выводят. Для понимания формулы нужно понять и способ вывода, а так же почему этот способ верный.


    1. plutarh
      30.10.2023 17:26

      Расстояние между двумя кораблями будет увеличиваться быстрее скорости света, да. Скорость, как мера изменения расстояния за время, вообще-то не ограничена. Ограничена и конечна скорость перемещения именно вещества (информации). По крайней мере, так я себе это понимаю.


      1. Pshir
        30.10.2023 17:26
        +3

        Именно. Во вращающейся системе отсчёта всё ещё интереснее. Можно ничего не представлять, а посмотреть на небо, и увидеть, как галактика Андромеды, находящаяся на расстоянии 2.5 миллиона световых лет от нас делает оборот по небу за одни сутки. То есть, для стоящего на Земле наблюдателя, она летает вокруг него со скоростью почти в 6 триллионов раз превышающей скорость света :)


      1. ilriv
        30.10.2023 17:26
        +3

        И вот тут мы получаем мнимый парадокс, который невозможно понять, рассматривая явления в 3-хмерном пространстве.

        Представьте что мы одновременно запускаем два фотона: один фотон с Северного полюса Земли, другой с Южного. Они летят в противоположных направлениях со скоростью света. Оба фотона удаляются от Земли со скоростью света. А значит скорость фотонов друг относительно друга должна быть равна 2*с. Но скорость фотонов друг относительно друга не может превышать скорости света. Парадокс?

        Парадокс существует, только если мы рассматриваем движение в 3-хмерном пространстве. А в 4-хмерном пространстве никакого парадокса нет.

        Красная линия соответствует телам, которые движутся со скоростью меньше скорости света. Синяя линия - частицам, движущимся со скоростью света. Теперь смотрите на конус: фотон, движущийся направо, удалится от начала координат на расстояние c*t, фотон движущийся налево, удалится тоже на расстояние c*t. При этом расстояние между двумя фотонами тоже будет равно c*t (а не 2*c*t, как должно было получиться по законам геометрии в трехмерном пространстве). Расстояния высчитываются как в равностороннем треугольнике.


        1. MishaRash
          30.10.2023 17:26

          Но скорость фотонов друг относительно друга не может превышать скорости света. Парадокс?

          Что-то тут не то... Начиная с того, что у отдельного фотона собственное время вообще не идёт и привязанная к нему отсчёта имеет от этого ряд проблем, поэтому в специальной теории относительности точка зрения фотона не применяется.


        1. Merrynose
          30.10.2023 17:26
          +1

          Не совсем понятно, как на картинке изменяются расстояние между фотонами: если как проекцию синей линии на ось Х, то вроде как получается расстояние 2*с*t, а не с*t. Можно попросить вас пояснить этот момент?


    1. Rishquer
      30.10.2023 17:26
      +3

      Непонимаю, за что заминусили. Человек задал нормальный вопрос по теме, на который не знает ответа.


  1. onets
    30.10.2023 17:26
    +1

    Таки в итоге - если лететь со скоростью 0.9999999999с, то можно пролететь весь Млечный Путь за месяц/год и не надо никакой гибернации для экипажа?


    1. Tzimie
      30.10.2023 17:26
      +5

      Не надо. Но ваши родные умрут. Кроме того, очень тяжело лететь, газ все норовит ионизировать, а любая песчинка - убить


      1. yar3333
        30.10.2023 17:26
        +10

        Даже не только газ. Реликтовое излучение, заполняющее вселенную, из ооочень длинных волн сместится в фиолетовую область и станет каким-нибудь рентгеновским излучением или гамма-лучами, от которых очень трудно защититься.


    1. tsvetkovpa
      30.10.2023 17:26
      +5

      Ну за месяц по часам экипажа.

      Для всей остальной галактики они будут лететь с околосветовой скоростью и потратят несколько десятков тысяч лет на пересечение галактики


      1. p-oleg
        30.10.2023 17:26

        А с точки зрения наблюдателя на корабле, вся остальная галактика летит с околосветовой скоростью?


        1. VBDUnit
          30.10.2023 17:26
          +6

          С точки зрения наблюдателя на корабле галактика будет сплющена вдоль направления движения и время в ней будет ускорено. Условно говоря, галактика толщиной 100 км и вся в быстрой перемотке. Звезды как диски. Плюс эффект Доплера и ещё куча всего.

          С точки зрения галактики корабль летит десятки тысяч лет, он сплющен (не растянут) вдоль своей траектории, и время на нем ускорено (не замедлено), т. е. в быстрой перемотке. Да, это контринтуитивно. Нет, это не противоречит логике, хотя выглядит так, как будто бы да.


          1. p-oleg
            30.10.2023 17:26
            +1

            Если галактика летит относительно корабля с околосветовой скоростью, почему время замедляется в корабле, а не в галактике, с точки зрения корабля?


            1. p-oleg
              30.10.2023 17:26

              П.С. А, вы долнили свой ответ, пока я писал вопрос.


            1. Tyusha
              30.10.2023 17:26

              Всё относительно. С точки зрения галактики наоборрт корабль будет сплющен, и именно на корабле время будет идти замедленно.

              Но с корабля жителям галактики ответят: да нет же, у нас всё нормально и с линейкой, и с часами, это у вас наоборот время идëт медленно, и вы сплюснутые, а не мы.

              И те, и другие будут правы. А ещё один летящий навстречу корабль скажет: вы оба сплюснутые и тормознутые, галактика ещё более менее, а первый корабль совсем сильно.


              1. MishaRash
                30.10.2023 17:26
                +2

                Стоит помнить, что в специальной теории относительности система отсчёта — это немного гипотетическая сущность, т.к. она требует наличия синхронизированных часов в любой точке пространства, движущихся с одной скоростью (а иначе их и нельзя синхронизировать). Скорость всех этих часов и определяет систему отсчёта.

                Когда мы говорим "с точки зрения галактики время на корабле будет идти замедленно", подразумевается, что часы на корабле сравниваются с часами на скорости галактики в двух разных точках и разность их показаний больше, чем разность показаний часов на корабле.

                Когда мы говорим "с точки зрения корабля время в галактике будет идти замедленно", подразумевается, что часы в какой-то фиксированной части галактики сравниваются с часами на скорости корабля в двух разных точках (они не могут быть обе на корабле) и разность их показаний тоже больше, чем разность показаний часов в той части галактики.

                Когда мы говорим, что с точки зрения галактики корабль сплющен, подразумевается, что его длина как расстояние между двумя часами на скорости галактики, мимо которых проходят передний и задний конец корабля при неких одинаковых показаниях этих часов.

                Часто в реальных измерениях наблюдатель лишь один и получает информацию с другого, движущегося относительно него объекта с переменной задержкой, что требует дополнительного расчёта и приводит к другим результатам в общем случае.


                1. MishaRash
                  30.10.2023 17:26

                  Когда мы говорим, что с точки зрения галактики корабль сплющен, подразумевается, что его длина определяется/измеряется как расстояние между двумя часами на скорости галактики, мимо которых проходят передний и задний конец корабля при неких одинаковых показаниях этих часов.


              1. p-oleg
                30.10.2023 17:26

                Если с точки зрения галактики (Земли) время замедлено на корабле, а с точки зрения корабля время замедлено на Земле, то что будет, если корабль вернется на Землю? Встретятся те же люди что и на старте или нет?


                1. kipar
                  30.10.2023 17:26
                  +3

                  пока корабль летит с постоянной скоростью нет различия - то ли это корабль пролетает сквозь галактику, то ли обитатели галактики пролетают мимо на своей галактике, а корабль стоит. Но в моменты когда корабль будет разгоняться и потом затормозит, его время будет меняться относительно галактики. Ну и когда он окончательно остановится станет ясно что те кто на корабле живы а в галактике прошли тысячи лет.

                  Ну, как ясно - корабль прилетел на звезду удалённую на 10к лет. Но при этом может уловить сигналы с земли, отправленные через месяц после отправления корабля. Так что можно сказать "а на земле тоже времени не прошло", но вот вернутся туда или отправить весточку раньше чем за 10к лет по её времени не выйдет.


          1. IvanPetrof
            30.10.2023 17:26

            галактика будет сплющена вдоль направления движения

            В связи с этим меня постоянно посещает мысль, что таким образом можно повысить угловое разрешение телескопа.

            Берём телескоп и смотрим вдаль на две звезды, угловое расстояние между которыми, телескоп не может разрешить (звёзды сливаются). Но если разогнать телескоп в направлении этих звёзд, пространство в этом направлении "сплющится" и эти две звезды "разъедутся" на угол достаточный, чтобы их наблюдать раздельно.

            Бонусом получаем фиолетовое смещение и можем наблюдать далёкие галактики в более обычном диапазоне, чем инфракрасный.

            Что тут не так? (ну кроме вопроса - как разогнать телескоп до релятивистских скоростей))


            1. Tyusha
              30.10.2023 17:26
              +5

              Нет, не так. Если разогнать телескоп, то в направлении движения наоборот звезды будут собираться к оси движения, а если смотреть назад, то да, сзади они будут разъезжаться по сфере.

              Но если даже так сделать и назад, тот повысить разрешение телескопа не получится, т.к. определяется диффракционным пределелом. Да, звезды разойдутся на бОльшее угловое расстояние, но за счёт красного смещения диффракционным предел станет настолько же хуже. Короче говоря, шило на мыло.


              1. IvanPetrof
                30.10.2023 17:26

                Интересная идея, если разогнать назад, то можно рентгеновский спектр перевести в оптический, под который у нас есть оптика.


              1. MishaRash
                30.10.2023 17:26

                Да, с диффракционным пределом обидно, упустил. В принципе, из-за конструктивных сложностей/дефектов разрешение может быть ограничено снизу выше диффракционного предела, но такая ситуация нежелательна (особенно в космосе, т.к. на Земле ещё и нестабильность атмосферы ограничивает существенно).

                Более того, телескоп начнёт получать меньше фотонов от источника за единицу времени (по своим часам). И от Земли он рано или поздно будет удаляться с большой скоростью, из-за чего интервал между приёмами сигналов с него на Земле будет тоже больше, чем интервал между их отправками по часам телескопа. Т.е. потребуется значительно больше времени для накопления такого же числа событий. Кстати, в рентгеновских (или гамма?) наблюдениях как раз часто очень мало фотонов набирается, каждому практически своё имя дают...


            1. MishaRash
              30.10.2023 17:26
              +1

              То, что вы описали — известный эффект, называемый релятивистской аберрацией. Формула для синусов даже красивее:

              \sin\theta_o = \frac{\sin\theta_s}{\gamma (1-\beta \cos \theta_s)},

              где

              \beta \equiv \frac{v}c, \quad \gamma \equiv \left(1-\beta^2 \right)^{-1/2}.

              При околосветовых скоростях (соответственно больших гаммах) и малых углах относительно направления движения вообще можно показать

              \theta_o \approx \frac{2\gamma}{1+\gamma^2\theta_s^2} \theta_s,

              т.е. для совсем малых углов можно достичь увеличения в 2\gamma.

              UPD: Действительно, я тут сначала запутался с направлениями. Полезно помнить, что поперечный (скорости относительного движения) компонент волнового 4-вектора не меняется и равен \omega \sin\theta (возможно, с делением на c, но это не так важно). Т.е. при фиолетовом/голубом смещении, которое возникает при сближении источника и наблюдателя, уменьшается длина волны, растёт частота и уменьшаются углы. Для увеличения нужно красное с разбеганием. Спасибо @Tyusha за поправку.

              Но в принципе схема для повышения разрешения рабочая, осталось только довести телескоп до околосветовой скорости, чтобы он не сломался при разгоне и его не пробило какой-нибудь частицей или излучением. Кроме того, выгодно будет наблюдать только около направления, противоположного скорости, потому что в других направлениях угловые размеры будут сжиматься (особенно против скорости). И дополнительное красное смещение усложнит наблюдения.


          1. Tyusha
            30.10.2023 17:26
            +2

            Здрасте, приехали. Время в сплющенной галактике с точке зрения корабля будет не ускорено, а наоборот замедлено.


        1. aamonster
          30.10.2023 17:26
          +1

          О, вы вплотную подошли к парадоксу близнецов :-).

          Да.


    1. MaximRV
      30.10.2023 17:26
      +1

      Более того, для фотонов - время несуществует. Ибо они летят Со скоростью света.


      1. CTheo
        30.10.2023 17:26
        +1

        Где-то читал, что это значит, если бы у фотона были глаза, то он бы "видел" все свое путешествие по маршруту одномоментно, и начало, и середину, и конец пути.


        1. Survtur
          30.10.2023 17:26

          Мне легче думать об этом как о пространстве которое с точки зрения фотона сплющится в ноль и он будет сразу и в начале, и в конце. Для него пути нет. Есть одна точка его положения, в 3Д вселенной сплющенной до 2Д.


          1. ksbes
            30.10.2023 17:26

            Не 3D, а 4D - времени-то для него тоже не будет.
            Но вообще при попытке посмотреть "с точки зрения фотона" неизбежно приходишь к противоречию. Например, т.к. время "сплющено" в ноль, то каждый фотон должен "видеть" начало и конец Вселенной. Хотя с нашей, медленной точки зрения - фотоны по крайней мере в начале не существовали и "видеть" это начало не могли. Уж фотоны из моей лампочки - точно.
            И это не говоря уже о том, что когда мы говорим "фотон" - то поднимает голову квантовая теория. А она с такой 2D вселенной не дружит.


            1. PrinceKorwin
              30.10.2023 17:26

              Не 3D, а 4D - времени-то для него тоже не будет.

              Почему 4D? Время для фотона отсутствует. Это 4-е изменение для него можно считать схлопнутым/свёрнутым. Нет?

              точно.

              И это не говоря уже о том, что когда мы говорим "фотон" - то поднимает голову квантовая теория. А она с такой 2D вселенной не дружит.

              А как вы тут от 4D к 2D перешли?


              1. ksbes
                30.10.2023 17:26

                Ну из четырёх измерений -x,y,z,t осталось только два x,y. А z и t Вселенной "схлопнулись".

                А как вы тут от 4D к 2D перешли?

                Ну дык, мы ж говорим о вселенной "с точки зрения фотона" - а значит и "с точки зрения фотона" пытаемся рассмотреть квантовый процесс его испускания/поглощения/запутанности с другим фотоном ...


                1. PrinceKorwin
                  30.10.2023 17:26

                  Но вы же сказали сами, что:

                  Не 3D, а 4D - времени-то для него тоже не будет

                  Почему 4D если времени не будет?

                  И почему Z схлопнулась?


                  1. delimer
                    30.10.2023 17:26

                    Точнее не z схлопывается, а траектория фотона. Так, как для фотона времени нет, то он находится одновременно во всех точках своей траектории.


    1. VBDUnit
      30.10.2023 17:26
      +3

      Да, можно без гибернации. Даже доширак завариться не успеет, уже пролетите. Серьёзно. Для Вас это путешествие займет очень мало времени, может даже, 5 секунд. Это что будете наблюдать Вы. А с точки зрения Земли Вы будете лететь со скоростью 0,99 999 999 с в течение десятков тысяч лет.

      С точки зрения фотона (если так можно выразиться), например, мир вообще двумерен: его путь равен 0, его время полёта равно 0. Измерение времени отсутствует, измерение, вдоль которого он летел, тоже. Для наблюдателя же фотон вполне себе летит со скоростью света. Именно поэтому, даже теоретически, не получится «лететь со скоростью света и включить фонарик» — Ваше время — это точка, а не линия. Действия делать не получится, когда в распоряжении лишь одно мгновение.


      1. MaximRV
        30.10.2023 17:26

        Про дошик вы это зря. Разгонять и тормозить корабль нужно время, и наверное, довольно большое время. Так что завариться он скорее всего успеет, и много раз.


        1. VBDUnit
          30.10.2023 17:26
          +2

          Ну так ускоряться можно совсем-совсем быстро :) Плюс при хорошем ускорении сжатие доширака повысит его давление, температуру и скорость заваривания. И приправа лучше впитается


          1. CBET_TbMbI
            30.10.2023 17:26

            Быстро - нельзя. Раздавит. А если разгоняться с ускорением в 1 g, то по понадобится порядка года.


            1. VBDUnit
              30.10.2023 17:26
              +2

              Ну если у нас задача не только заварить дошик, но ещё и выжить, то можно задействовать диамагнетическую левитацию, чтобы скоменсировать ускорение.

              Генерируем магнитное поле в 10 000 Тл и отталкиваем тушку в сторону, противоположную ускорению. Поскольку магнит действует на все точки тела, а не на её поверхность, силу, действующую на тело, можно будет снизить до 1 g. При этом, магнитное поле должно быть не просто сильным, а устроено хитро, чтобы не было приливных сил. Но если вы построили штуку, которая летает 0,999 999 999 с, то это не есть проблема, скорее всего.


              1. CBET_TbMbI
                30.10.2023 17:26

                Сомневаюсь, что любые меры могут компенсировать ускорение. Ну, разве что совсем фантастические типа искривления пространства, антигравитации т.п. Насчёт магнита сомневаюсь, что можно такой создать. Нужно чтобы он действовал одинаково на каждую молекула тела. Это при том, что магнит в принципе не оказывает влияния на органику.


                1. VBDUnit
                  30.10.2023 17:26

                  В том то и фокус, что оказывает. Речь не о магнитной левитации, а о диамагнетической. Диамагнетические материалы не «не магнитятся», они выталкиваются магнитным полем, просто этот эффект очень слабый.

                  Лягушка в магнитном поле >10 тесла, с ней всё ок
                  Лягушка в магнитном поле >10 тесла, с ней всё ок

                  И магнитное поле оказывает влияние именно на каждый атом. Главное, чтобы поле было равномерное, это как бы сложно, но с лягушкой же получилось. В обычных условиях, если положить маленькую графитовую пластинку на неодимовый магнит, она начнёт парить — по той же причине.

                  Поэтому, имхо, можно использовать диамагнетическую левитацию для компенсации ускорения.


                  1. CBET_TbMbI
                    30.10.2023 17:26
                    +1

                    хмм... Не знал. Но всё равно сомнительно насчёт безопасности. Разные же молекулы по разному могут реагировать. В той же крови железа больше, чем, например, в костях.


                    1. VBDUnit
                      30.10.2023 17:26

                      Тут да, надо считать. Однако, имхо, до определённого предела это работать будет, пока атомы железа не вырвет из молекул гемоглобина:)

                      Если этого будет мало — создать искусственное гравитационное поле в передней части корабля, чтобы это гравитационное поле, грубо говоря, притягивало тело вперёд так, чтобы компенсировать всё ускорение, за исключением 1g.


      1. Hlad
        30.10.2023 17:26

        Маловато девяток :) При движении с такой скоростью время замедлится всего примерно в десять тысяч раз, то есть, секунда будет соответствовать 2-3 часам с точки зрения Земли.


    1. vanxant
      30.10.2023 17:26
      +4

      Если иметь корабль, способный поддерживать постоянное ускорение 1g, можно метнуться в Галактику Андромеды и обратно меньше чем за год. Но есть нюанс: на Земле пройдут миллионы лет.


      1. Taus
        30.10.2023 17:26

        А за жизнь можно успеть слетать к границе видимой Вселенной и обратно ;)


        1. Tyusha
          30.10.2023 17:26

          При постоянном ускорении зависимость расстояния путешествия от собственного времени экипажа корабля — это экспонента при субсветовых скоростях (гиперболический косинус). Поэтому улететь можно куда угодно.


          1. PrinceKorwin
            30.10.2023 17:26

            При постоянном ускорении

            А как это обеспечить если для ускорения приходится расходывать рабочее тело, т.е. массу?


            1. ksbes
              30.10.2023 17:26

              Не обязательно - у нас поля есть: гравитационное и и электромагнитное. Например, светим очень ярко кораблю в попу - он и ускоряется от светового давления. Постоянство обеспечиваем подкручиванием яркости


              1. PrinceKorwin
                30.10.2023 17:26

                Проблема фокусировки и рассеяния не позволит вам бесконечно толкать корабль.


              1. Pshir
                30.10.2023 17:26

                Экспоненциальное подкручивание яркости - это очень круто


          1. Taus
            30.10.2023 17:26

            Могло бы не хватить времени жизни человека. Но так совпадает, что 1g хватит.


      1. ykira
        30.10.2023 17:26

        Вопрос к знатокам, если с 1g это год, то с 2g это пол года, четверть года, или другое число?


        1. vanxant
          30.10.2023 17:26

          Для СТО - четверть года. А так, конечно, это всё не очень научная фантастика:)


      1. DBalashov
        30.10.2023 17:26
        +1

        Тау Ноль by Пол Андерсон


    1. sargon5000
      30.10.2023 17:26

      Есть голубая звезда, Джанетта,
      Езды до нее пятнадцать лет,
      Если мчаться со скоростью света.

      И белая есть звезда, Джанетта.
      Езды до нее сорок лет,
      Если мчаться со скоростью света.

      К какой же звезде
      Мы с тобой поедем —
      К голубой или белой?

      Сорри за оффтоп. Навеяло.


    1. Wizard_of_light
      30.10.2023 17:26
      +3

      При v=0.9999999999 с лоренц-фактор составит 70710,6. Если диаметр Млечного пути принять за 100 000 световых лет, то путешествие от края до края займёт примерно 17 месяцев по собственному времени. До ближнего края Туманности Андромеды с такой скоростью придётся лететь примерно 34 года по собственному времени. Кинетическая энергия каждого килограмма корабля будет энергетическим эквивалентом 70,7 тонн. Температура реликтового излучения, светящего спереди по курсу, составит ~1106 К.


      1. IvanPetrof
        30.10.2023 17:26
        +1

        Благодарю вас мистер Спок! Мистер Чехов, курс на Андромеду!


    1. Vold2D
      30.10.2023 17:26
      +1

      Есть такой старый фантастический роман - "Тау ноль". Там подобная ситуация неплохо описана.


      1. Mingun
        30.10.2023 17:26

        Роман, это вы загнули, конечно. Рассказик.


        1. Vold2D
          30.10.2023 17:26

          Две сотни страниц. Вполне роман.


          1. Mingun
            30.10.2023 17:26

            Всё же поменьше 200 страниц будет. На телефоне читалка показывает 320 страниц, но там такие страницы… на пять-десять предложений. И всё-таки вы правы, как оказалось, текста прилично. Видимо, я так проглотил его на одном дыхании, что даже и не заметил, вот он мне и показался коротким :)


    1. bbs12
      30.10.2023 17:26

      Если лететь со скоростью частицы OMG, то с Земли до галактики Андромеды - 5 минут по часам путешественника.


      1. IvanPetrof
        30.10.2023 17:26

        ...а с гравицапой в любую точку вселенной - за пять секунд..


  1. kauri_39
    30.10.2023 17:26
    -3

    Скорость, с которой движутся фотоны, будет такой же, как и всегда, - скоростью света, причём не только с вашей точки зрения, но и с точки зрения любого наблюдателя.

    А если фотон движется в гравитационном поле, где ход времени замедляется? В книге Жарова "Сферическая астрономия" упоминается о "гравитационном замедлении фотонов". Или там, в римановом пространстве, расстояния тоже уменьшаются, и поэтому скорость света остаётся неизменной?


    1. VBDUnit
      30.10.2023 17:26
      +2

      Для фотона его путь и время существования равны 0, скорость равна 0/0. Для стороннего наблюдателя он всегда движется со скоростью света, но может менять частоту и лететь по кривой, в случае, если эта кривая — кратчайший путь между точкой испускания и поглощения фотона.


    1. MishaRash
      30.10.2023 17:26

      С одной стороны, в общей теории относительности допустимы почти любые координаты, так что через их переопределение можно получать совершенно разные скорости света.

      С другой стороны, когда встанет вопрос о физических измерениях, всплывёт то, что у нас эталон расстояния выражается через эталон времени и скорость света, а эталон времени определён через частоту определённого излучения, которое идёт по собственному времени.

      В метрике Шварцшильда (невращающаяся чёрная дыра) и "обычных" координатах ни время, ни радиус не "физичны". То есть в точке с постоянными пространственными координатами ( r,\theta,\varphi ) интервал собственного времени d\tau не равен интервалу координатного времени dt. Также для света, движущегося радиально, \left| dr/dt \right| \ne c . Но зато координатное время синхронизировано в том смысле, что световые сигналы, между испусканием которых из одной точки в пространстве в одинаковом направлении прошло \Delta t, будут получены в другой точке на их общем пути тоже с интервалом координатного времени \Delta t .


      1. kauri_39
        30.10.2023 17:26

        Спасибо, я и забыл, что у нас эталон длины, применяемый для расчёта скорости света, ею же и определяется. Это нормально? Сама скорость света определяется длительностью секунды, а секунда определяется частотой фотонов в цезиевых часах. Из Вики:

        "Метр есть длина пути, проходимого светом в вакууме за интервал времени 1⁄299 792 458 секунды. Секунда — время, равное 9 192 631 770 периодам излучения, соответствующего переходу между двумя сверхтонкими уровнями основного состояния атома цезия-133."

        Хотелось бы разобраться с физическим смыслом скорости света, опираясь на физические понятия, а не на геометрические, где допустимы "почти любые координаты" и "разные скорости света". Возьмём за основу число периодов излучения - частоту фотонов, излучаемых цезием в атомных часах.

        Известно, что за земную секунду излучение цезия на орбите спутников gps имеет на 5 периодов больше, то есть частота излучаемых им фотонов равна 9192631775 Гц. Если в этом случае считать секундой земное число периодов, то космическая секунда по своей длительности будет короче земной, и время в космосе идёт быстрее.

        Это в принципе соответствует ОТО, которая говорит о замедлении времени в гравполе и о его ускорении вдали от него. Так и есть в реальности: в ход времени часов на спутниках gps вводят замедляющую поправку для их синхронизации с земными часами.

        Но тогда и космический метр, определяемый прохождением света за за более короткий космический интервал времени, должен быть чуть короче земного. Но если в реальности это не так, если метр по своей длине в космосе остаётся как минимум неизменным, то тогда скорость света в космосе должна быть чуть больше, а в гравитационном поле - меньше. Есть ли в этих рассуждениях логическая ошибка?


        1. MishaRash
          30.10.2023 17:26
          +1

          Тут вопрос, как сравнивать интервалы времени и длины в разных точках.

          Для времени можно смотреть, как меняется временной интервал между двумя сигналами, отправленными по одной пространственной траектории, как я писал выше, или период световой волны, что практически то же самое. Тогда получается, что глубже в гравитационном потенциале время идёт медленнее в том смысле, что между отправками сигнала в "глубине" успело произойти меньше процессов с фиксированной физической частотой (например, спонтанное излучение атома в определённом возбуждённом состоянии), чем между получениями ближе к "поверхности". Если сигналы были отправлены в обратном направлении, ближе к "поверхности" потенциальной ямы между ними всё равно произойдёт больше таких же локальных физических процессов, чем между их принятием в "глубине".

          Вы именно такую синхронизацию неявно подразумеваете, когда применяете "за земную секунду" к спутникам GPS. А можно ещё измерять собственное время в каждой точке, т.е. считать количество периодов излучения локальных атомов цезия и делить на 9 192 631 770 по определению секунды. Так получится, что между синхронизациями на спутнике прошло чуть больше собственного времени, чем на Земле.

          Обычно время в метрике Шварцшильда — именно синхронизированное время, которое совпадает с собственным временем только бесконечно далеко от чёрной дыры, а в любой точке на конечном расстоянии (над горизонтом) собственное время идёт медленнее. Под горизонтом есть проблема с определением такого синхронизированного времени, потому что из-под него ничто выйти не может.

          Для длины можно использовать скорость света, или длину световой волны (с чем вроде как практически то же самое получится). Длина волны будет увеличиваться по мере "выкарабкивания" из потенциальной ямы. Длина волны такого же излучения (скажем, фиксированной спектральной линии), "произведённого" на месте, будет меньше, чем для волны, пришедшей "из глубин".

          Если такой подход для расстояния вас не устраивает, то вопрос — а как иначе? Раньше эталоном метра была жёсткая палка, которую приходилось хранить в строго контролируемой нейтральной атмосфере для предотвращения окисления, испарения и прочего. Другая причина, почему это не хороший вариант — в действительно сильных гравитационных полях, когда все эти эффекты замедления времени особенно заметны, окажется, что пределы жёсткости и даже прочности большинства известных материалов не настолько велики, чтобы можно было пренебрегать сплющиванием/растягиванием.


          1. kauri_39
            30.10.2023 17:26

            Вот и получается, что если длину измерять скоростью света, то саму скорость света такой длиной точно не измерить. Судим по аналогии: 1 метр есть длина пути, проходимого звуком в воздухе за интервал времени 1/343 секунды. В более плотном воздухе, где скорость звука выше (но мы считаем её постоянной), метр окажется длиннее, чем измеренный в менее плотном воздухе. Теперь измерим скорость звука в плотном воздухе. В реальности она выше, но звук проходил там по "длинным метрам", поэтому расчётная скорость оказалась одинаковой с той скоростью, которая рассчитывалась для звука, проходившего в менее плотном воздухе по "коротким метрам". Получили подтверждение постоянства скорости звука.

            А ведь вакуум - это энергетически плотная среда, и плотность её энергии наверняка разная на поверхности массивного тела и вдали от него. Электромагнитные волны поперечные, в отличие от звуковых, но вдруг и у них есть зависимость скорости распространения от свойств физического вакуума?

            Можно вообще рассматривать фотон как точечную частицу - как квант материи, перемещающийся в плотной среде квантов вакуума, стремящихся к расширению. Перемещение является следствием постоянной ликвидации фотоном квантов вакуума, стоящих у него на пути и давящих на него. Тогда энергия фотона будет определяться числом квантов вакуума, ликвидируемых в единицу времени. Скорость его перемещения постоянна в вакууме с неизменной плотностью энергии (с космологической постоянной). А вот в гравитационном поле возможны "координатные аномалии".

            Чтобы выйти на известную скорость света, надо знать размеры квантов вакуума, а они могут быть во много-много раз меньше планковской, и скорость ликвидации каждого кванта. Но в этом мало толку, пока мы не научимся определять плотность энергии вакуума в любой точке пространства. Может, её несложно определить по величине Лэмбовского сдвига электронов? "Старый Мазай разболтался в сарае..."


            1. MishaRash
              30.10.2023 17:26

              Вот и получается, что если длину измерять скоростью света, то саму скорость света такой длиной точно не измерить.

              При таком выборе эталонов скорость света вообще по сути не измерить, она всегда получится равной принятой по определению. А координаты в ОТО можно перевести в физические величины с помощью метрики.

              Аналогия со звуком в воздухе не очень удачна потому, что там есть естественная система отсчёта, в которой молекулы среды (воздуха) в среднем покоятся, и соответственно относительно неё две отдельные скорости, источника и приёмника.

              У света же нет выделенной среды с определённой скоростью, как показал опыт Майкельсона-Морли ещё тогда, когда эталоном длины был стержень. Поэтому важно только относительное движение источника и приёмника (одна скорость).

              Вы теперь предлагаете в качеству фундаментальной длины размер квантов вакуума из вашей философской теории, где на словах получаются все известные результаты, да ещё и практически все проблемы современной физики якобы решаются, нужен лишь физик, который воодушевится и додумает за вас самую малость — как же эту всю прелесть обсчитать. Кроме того,

              • почему у этих квантов вакуума вообще должен быть определённый размер?

              • как вы собрались измерять масштабы меньше планковской длины?


              1. kauri_39
                30.10.2023 17:26
                -1

                Если исходить из эквивалентности инертной и гравитационной массы, то у света есть выделенная система отсчёта: неподвижный физический вакуум, в котором нет гравитационных полей. Когда тело ускоряется сквозь него и относительно него, оно приобретает вес. И когда это тело неподвижно, а сквозь него с тем же ускорением движется вакуум, втекающий в массивное тело (в источник гравитации), то оно приобретает тот же вес.

                Поэтому с оценкой опыта Майкельсона-Морли получился конфуз. Ну не нашли они эфирного ветра в горизонтальной плоскости, но это же не означает, что эфира нет вообще. А если его "ветер дует" в вертикальной плоскости, и если это он сообщает одинаковое ускорение свободно падающим телам разной плотности? Ньютон ведь не только подтвердил это на опыте с падением пера и дробины в трубке с откачанным воздухом, он даже предположил природу гравитации как давления внешнего, более плотного эфира на тела, в которых плотность эфира меньше ("Оптика", 3 книга, 21 вопрос). Почему все об этом забыли - непонятно.

                И с теориями Эйнштейна вышло недоразумение. СТО обходится без эфира, но для ОТО он нужен. Что будет обеспечивать стационарность Вселенной (ныне - её ускоренное расширение) как не среда между телами, противостоящая их взаимному гравитационному притяжению? В итоге теперь эту среду называют "физическим вакуумом" или "тканью пространства-времени" - в назидание будущим учёным. Хорошо хоть не стесняются признавать, что гравитационные волны зарегистрированы с помощью интерферометра Майкельсона.

                Я полагаю, что перед созданием работающей теории квантовой гравитации физикам нужно предварительно откатиться в философию и наконец-то определить природу гравитации: выявить процесс, в результате которого материя искривляет пространство-время или, говоря иначе, снижает плотность физического вакуума. Если уже сформировалась такая потребность, то возрастает спрос на философскую модель мира, которая создана по научному методу и где объясняется природа гравитации. Тогда мой пятимерный мультиверс ещё пригодится.

                Кванты вакуума должны иметь некоторый средний размер, как имеют его вселенные нашего масштаба пространства-времени, расширяющиеся навстречу друг другу. Эти вселенные - будущие кванты вакуума и кванты материи (фотоны) во вселенной следующего за нашим масштаба. Поэтому кванты нашего вакуума - это уже безжизненные вселенные предыдущего нам масштаба. Как измерить размер таких квантов? Пока лишь подгонкой под результаты процессов с их участием. То есть под известную скорость света, которая есть следствие вышеописанного перемещения фотона в среде квантов вакуума.


                1. MishaRash
                  30.10.2023 17:26

                  А если его "ветер дует" в вертикальной плоскости, и если это он сообщает одинаковое ускорение свободно падающим телам разной плотности?

                  Это, мягко говоря, необычный ветер, который сообщает только ускорение, не влияя на саму скорость. Насколько я знаю, из механистических соображений (как у Ньютона) его не получилось описать без противоречия с экспериментальными результатами. Кроме того, искривление пространства в ОТО это хорошо описывает, чем ваши идеи лучше? Хорошо, вы спускаетесь на уровень глубже (что тоже, пожалуй, не всегда обязательно к лучшему), но в чём практическое преимущество, если всё только на словах, а численно получаются только уже известные результаты (через существующие теории, принципы которых следуют из вашей фундаментальной парадигмы)?

                  Что будет обеспечивать стационарность Вселенной (ныне - её ускоренное расширение) как не среда между телами, противостоящая их взаимному гравитационному притяжению?

                  Если (вам) известно только одно объяснение, это не значит, что оно правильное и полезное. Проблемы современной физики — в довольно специфичных деталях: плотность вакуума из (известной части) квантовой теории поля не сходится с космологической постоянной; в расчётах квантовой гравитации на высоких энергиях не убираются бесконечности и т.д.

                  Кванты вакуума должны иметь некоторый средний размер, как имеют его вселенные нашего масштаба пространства-времени, расширяющиеся навстречу друг другу.

                  Какой размер вселенных вы имеете в виду? По текущим моделям и измерениям, наша Вселенная может быть и бесконечной, хотя в ней есть масштабы, определяемые конечным временем её существования или скоростью расширения.


                  1. kauri_39
                    30.10.2023 17:26
                    -1

                    Почему же ускоренное движение вакуума "сообщает только ускорение, не влияя на саму скорость"? Например, в гравитационных волнах его возвратно-поступательные движения приводят в такое же движение и материю, что выявляется во взаимно перпендикулярных плечах интерферометра Майкельсона. И тела, например, свободно падают на Землю, увлекаемые ускоренным потоком вакуума - вместе с вакуумом, поэтому пребывают в невесомости. Вакуум - это всё же не эфир времён Ньютона. Возможно, Лесаж исходил из таких же "механистических соображений", как у Ньютона, но дальше пошёл в ошибочном направлении - к созданию неверной "теневой" модели гравитации.

                    Да, ОТО хорошо описывает гравитацию как искривление пространства-времени, но только в том масштабе, где геометрия пространства под влиянием материи переходит от Евклидовой к Римановой. То есть где вакуум со средней плотностью энергии расширяется в сторону менее плотного вакуума. И не описывает искривление пространства в гораздо большем масштабе, где геометрия пространства под влиянием более плотного вакуума переходит от геометрии Лобачевского к Евклидовой. Это неучтённое ускоренное движение вакуума от более плотного к среднему значению - в войдах и между галактиками - приходится заменять дополнительной обычной гравитацией от необычной тёмной материи. А тот избыточный вакуум, что не втекает в галактики и вызывает разлёт кластеров, приходится обозначать тёмной энергией. Мои идеи проясняют эти тёмные сущности и упраздняют одну из них.

                    Не будем забывать, что ОТО вначале тоже выдавала "уже известные результаты" - известную величину смещения перигелия Меркурия и известные параметры вращения планет. Потом, моя теория философская, у неё особые предсказания, их сразу не проверишь, поэтому часть её предсказаний должна совпадать с известными результатами частных физических теорий (ОТО, КТП), чтобы подтвердить её верность.

                    В пятимерном мультиверсе плотность энергии вакуума выражается не через абстрактные виртуальные частицы, лишённые массы и энергии, а через плотные взаимно сжатые кванты вакуума - вселенные предыдущего масштаба пространства-времени. Образцом служит наша Вселенная - пока ещё свободно расширяющийся объём плотного вакуума. Об огромной плотности этой среды говорит тот факт, что при своём космологическом расширении и движении она сообщает одинаковое ускорение космическим объектам разной плотности. Из таких квантов вакуума могут рождаться реальные элементарные частицы, например, фотоны. Для этого нужно потратить определённую энергию, которая идёт на рождение новых квантов вакуума с большим временем свободного расширения, за которое эволюция материи внутри них (как в нашей Вселенной) успевает дойти до 8-й формы материи (единая вселенская система цивилизаций). Представители этой формы (разумные вселенные) являются фотонами во вмещающей их вселенной. Возможно, что таким путём получаются из вакуума фотоны в излучении Хокинга и фотоны под горизонтом ЧД. Последние идут на постоянный рост масс ЧД, который был предположен по результатам двух разных исследований (публикации 2021 г. и 2023 г.)

                    Вселенные, соразмерные нашей, могут превышать наш объём Хаббла в сотни миллиардов раз. Они расширяются навстречу друг другу со сверхсветовой скоростью. Я не математик и не предполагаю наличие дополнительных геометрических измерений. Для объяснения происхождения и будущего нашей Вселенной достаточно 5-го масштабного измерения. Его составляют бесчисленные масштабы вселенных, постоянно образующиеся из вселенных предыдущего масштаба. Косвенно 5 измерение прослеживается и в росте масштаба элементов новых форм материи (от кварков до цивилизаций и выше).


                    1. MishaRash
                      30.10.2023 17:26

                      Почему же ускоренное движение вакуума "сообщает только ускорение, не влияя на саму скорость"? Например, в гравитационных волнах его возвратно-поступательные движения приводят в такое же движение и материю, что выявляется во взаимно перпендикулярных плечах интерферометра Майкельсона. И тела, например, свободно падают на Землю, увлекаемые ускоренным потоком вакуума - вместе с вакуумом, поэтому пребывают в невесомости.

                      Как же вы определяете, когда материя следует скорости вакуума, а когда сообщается только ускорение? Извините, но пока больше всего похоже на выбор того варианта, что будет соответствовать известным феноменам, без зашитых в саму теорию критериев. Это, кажется, не единственный такой смутный аспект в вашей модели, но и один произвольный выбор в таком ключевом вопросе создаёт достаточно проблем. Мне кажется, это ещё и легче проглядеть в словесных рассуждениях, чем в расчётах.

                      Таким путём, конечно, можно легко объяснить все существующие эксперименты и наблюдения, но тогда и под большую часть новых придётся подгонять. А это уже и квантованная ОТО может, насколько я знаю, так как в ней с бесконечное количество калибровочных параметров в высокоэнергетическом режиме. Например, она не предсказывает плотность энергии вакуума конкретно, в её рамках это контрчлен, калибруемый на экспериментальных данных, и проблемы космологической постоянной будто бы и нет. При этом в низкоэнергетическом режиме, который содержит практически всё, доступное нам, у ОТО отличная предсказательная способность и ваша теория выглядит как существенное ухудшение.


                      1. kauri_39
                        30.10.2023 17:26

                        Материя полностью "следует скорости вакуума", когда она движется вместе с ним и покоится в нём, например, в свободном падении тел. При этом вакуум сообщает ей и своё ускорение, с которым он втекает в источник гравитации, расширяясь из космоса. Ничего "смутного" здесь нет. Тела могут двигаться и против того же гравитационного потока вакуума, испытывая перегрузку. Например, при движении тела по инерции или при реактивном движении.

                        В мою философскую теорию можно зашить только самые общие физические критерии. Например, среда - плотная, квантованная, стремится к расширению. Материя её разрежает, снижает её плотность внутри себя. Поэтому материя выглядит в среде как область большего (свинцовая дробина, нейтронная звезда, ЧД) или меньшего (птичье перо, газовая туманность) разрежения среды. Из этого следует, что любое движение среды приводит в такое же движение покоящуюся в ней материю разной плотности (областей с разной степенью снижения плотности среды).

                        Такие критерии не ведут к противоречиям с наблюдаемыми данными и дают возможность создать уже физическую теорию гравитации - без недостатков обычной и квантованной ОТО. Моя философская теория не может выглядеть "как существенное ухудшение" ОТО, потому что у этих теорий разные цели и средства их достижения. Больше, чем наполнение Вселенной тёмной материей из-за незнания природы гравитации, ухудшить ОТО ничто не может.


                1. flx0
                  30.10.2023 17:26

                  В итоге теперь эту среду называют "физическим вакуумом" или "тканью пространства-времени"

                  "Физический вакуум" и "ткань пространства-времени" - это слова, встречающиеся только в популярных "объяснениях" физики. Они там используются потому что обыватель не знает что такое операторы и многообразия. Когда обываететь потом считает, что он из этих популярных "объяснений" что-то понял, получается салат слов, как у вас. Про "кванты вакуума среднего размера" и прочие торсионные поля.


  1. Tarson
    30.10.2023 17:26

    Тут больше интересен другой вопрос. Если в звездолете масса космонавта увеличится из-за приближения к скорости света, допустим в 2-5 раз или даже больше, то как он себя будет чуйствовать и как будут идти в нем биологические процессы? Допустим, масса выросла в 5 раз, а постояннное ускорение звездолета 0,2 G , тогда вес будет эквивалентен земному mg. Но если космонавт стукнет другого космонавта, то импульс то в 5 раз больше будет. Ну и внутри космонавта все движущиеся части, от крови до еда будут иметь массу в пять раз большую же...


    1. VBDUnit
      30.10.2023 17:26
      +1

      Масса покоя — не есть масса релятивистская. Там все хитрее. Для космонавта его «масса» не поменяется, хотя поводов для физиологических проблем все равно будет море.


    1. bromzh
      30.10.2023 17:26
      +5

      Так в системе отсчёта корабля ничего не меняется


    1. Sap_ru
      30.10.2023 17:26
      +4

      С точки зрения космонавтов на борту из масса не изменится.


  1. DreamChild
    30.10.2023 17:26

    Дилетантский вопрос немного не по теме. Как известно все элементарные частицы одного вида (например электроны) абсолютно одинаковы и никак не отличаются друг от друга. Кроме того этих частиц очень много, квадриллионы квадриллионов. Как же тогда учёные понимают, что работают с одной и той же частицей ходе всего эксперимента? Например, гоняя электроны на десятки или сотни метров внутри ускорителя? Ведь в окружающей среде триллионы и триллионы точно таких же частиц.


    1. VBDUnit
      30.10.2023 17:26
      +1

      Как Вы узнаете, что Вам пожал только что руку именно Ваш коллега, а не его злой клон? Ну, маловероятно, что это злой клон. Так и тут: по признакам. Есть объект, у него есть ряд признаков. Например, заряд, масса, скорость, направление движения, ещё что‑нибудь. По ним этот объект идентифицируют. Если мы понаблюдали те же признаки, значит мы можем считать, что это тот же самый объект.

      Конечно, может быть это и не оно, а злой клон. Может это и не те самые частицы, а другие, и они все время рандомно телепортируются на место друг друга, и это и объясняет странности квантовых эффектов, но мы пока это померить и проверить никак не можем. Поэтому производим расчёты так, как будто бы это те самые как будто бы частицы. Расчёты сходятся с экспериментами — значит ок.


      1. DreamChild
        30.10.2023 17:26

        Но ведь частицы в отличие от людей и есть клоны друг друга. Коллегу я могу отличить по чертам лица, телосложению, голосу, походке, манере речи и десяткам других признаков. У частиц этих признаков совсем немного, и они зачастую одинаковы для всех объектов одного типа (возможно я ошибаюсь, но заряд и масса у всех электоронов равны). Да и коллег у меня не квадриллионы квадриллионов, а максимум десятки.


        1. VBDUnit
          30.10.2023 17:26
          +1

          частицы в отличие от людей и есть клоны друг друга

          А вот и не факт.

          Коллегу я могу отличить по чертам лица, телосложению, голосу, походке, манере речи и десяткам других признаков.

          Потому что можете. А если бы не могли? Если бы не существовало, не было открыто методов измерения телосложения, походки, манеры речи, черт лица? Как бы Вы считали — это все клоны или разные люди? Представьте мир, где Вы можете измерить только вес и рост человека, и больше ничего. Считали ли бы Вы, что все люди клоны? А почему бы и не нет? Или таки да? Вот‑вот:)

          Так и с частицами — мы не знаем, клоны они или нет. Мы умеем мерить вот это, это и это. Другое мы не знаем и мерить пока не умеем. Действительно ли это все признаки частиц, и других у них нет, и частицы в самом деле такие похожие, или же есть куча других признаков, которые мы пока просто не научились мерить — мы не знаем.

          Вот я пролистал страничку выше. Текст сместился. Как я определил, что это «тот самый текст» а не «точно такой же текст, возникший в новом месте»? Ну, это те же буквы, тем же цветом, тем же шрифтом, на том же примерно месте. Плюс я ожидал, что в результате моих действий текст окажется именно выше. Более того, в данном случае, половину «размышлений» сделал за меня мой организм в сетчатке глаза, распознавая движение «аппаратно». Поэтому я считаю, что это «тот самый» текст. А если у меня больше знаний, я, например, знаю, что такое пиксели, и что они на самом деле просто меняют цвет, а видеокарта заново рендерит картинку, я уже могу сказать, что вот, с такой‑то точки зрения это уже не «тот самый текст». И вообще тут движок браузера буфер пересоздал в видеопамяти. По сути, я сам решаю, что считать «тем самым», а что не считать.

          Вот тут примерно так же. Понятие «по‑настоящему те самые частицы» слишком условное и субъективное для науки, она прекрасно обходится без него — производит расчеты и предугадывает явления. Поэтому просто провозглашают — считать частицу «той самой», если она такая‑то и подчиняется таким то уравнениям. Всё.

          Даже «частица» — это лишь наше понятие о неком явлении/совокупности явлений. Что это «на самом деле», мы не знаем, но когда мыслим об этом, как о «частице», нам удобно, и у нас получается. Поэтому мы говорим, что вот это вот — «частица», и она «летит».


          1. DrSmile
            30.10.2023 17:26
            +1

            Частицы не просто клоны, они принципиально неразличимы, это строгий, экспериментально проверяемый квантомеханический факт. Если у нас были частицы в точках A и B, а стали — в точках C и D, то при расчете процесса необходимо комбинировать переход A→C : B→D вместе с A→D : B→C. Для бозонов комбинирование происходит со знаком плюс, для фермионов — со знаком минус (откуда и получается принцип Паули). Так что финальная частица в точке C — это некоторая суперпозиция исходных частиц из A и B.


            1. Tzimie
              30.10.2023 17:26
              +2

              Верно. И даже фотон, летя по прямой, временами может временами превращаться в виртуальную пару e+e- и снова рекомбинировать. Тот же ли это фотон? Вопрос больше условный, типа "живые ли вирусы"


              1. DreamChild
                30.10.2023 17:26

                Хорошо, пусть вопрос будет не "тот же ли это фотон?", а "как мы вообще понимаем, что наши опыты достоверны"? Как мы понимаем, что это не какой-то случайный фотон из триллионов таких же, не имеющих отношения к нашему эксперименту? Ведь в отличие от макромира мы здесь не можем ничего увидеть, потрогать, промаркировать и тд


                1. Tzimie
                  30.10.2023 17:26
                  +1

                  По повторяемости и предсказательной силе


        1. KurtkaBeyn
          30.10.2023 17:26

          Чтобы столкнуть частицы нужно поместить их в очень узкий пучок и разогнать до огромной скорости, чтобы при столкновении они не пролетели мимо друг друга. Всё таки очень большая сила отталкивания при сильном ядерном взаимодействии.

          Соответственно такие высокоэнергетические частицы существуют в коллайдере искусственно, в естественной среде у них нет таких энергий. Поэтому в коллайдере их можно отследить хотя бы примерно


    1. CBET_TbMbI
      30.10.2023 17:26
      +3

      Никак не различают. Но электроны, как и другие частицы, не летят из неоткуда. Все чувствительные экперементы проводятся в изолированных условиях. Часто в вакууме. Там левых частиц мизерное количество. А эксперементы проводятся многократно, чтобы исключить случайные события.


    1. ilriv
      30.10.2023 17:26

      Большинство экспериментов в элементарными частицами имеет статистический характер. То есть не пытаются различить отдельные частицы, а собирают статистику.

      Есть более тонкие эксперименты, где испускаются единичные частицы с определенным временным интервалом. Там уже понятно что регистрируется именно та частица, которую только что испустили.


      1. MishaRash
        30.10.2023 17:26

        Собственно, в квантовой физике все предсказания принципиально вероятностные, а не точные и конкретные, тут крайне сложно без статистики.


        1. ilriv
          30.10.2023 17:26

          Нет. Если вы испускаете один электрон в направлении цели и хотите посмотреть как он будет взаимодействовать с веществом, электрон попадёт в цель и будет как-то взаимодействовать. Это не вероятностное предсказание.


          1. ksbes
            30.10.2023 17:26
            +1

            А вот и нет. Есть вероятность, что протуннелирует и полетит дальше. Или вообще в мишень не попадёт. Или по пути создаст ещё электрон-позитронную пару.
            А ещё количество ваших частиц зависит от ускорения наблюдателя.


  1. funca
    30.10.2023 17:26

    Но если бы мы заменили пушку фонариком, история была бы совсем другой.

    Мы можем бросить пушку в озеро и вместо ядра выстрелить из нее водой, оценивая затем скорость расходящейся волны. Но что это даёт?


  1. Demon416
    30.10.2023 17:26

    Имхо теже эффекты которые даёт теория относительности будет давать вселенная с дискретным пространством-временем, но при этом не будет кучи парадоксов за ненадобностью.


    1. Wizard_of_light
      30.10.2023 17:26
      +2

      Это вы не пробовали задать дискретное пространство с наблюдаемыми свойствами. Парадокс возникнет уже в тот момент, когда в близкой к эвклидовой метрике придётся постулировать равный шаг сетки в произвольном направлении. Потом попробуйте сделать одинаковой скорость света для произвольного подвижного наблюдателя, при условии что она привязана к шагу сетки.


  1. wmlab
    30.10.2023 17:26
    -1

    Я для себя этот парадокс с фонариком на звездолете уяснил так: свет привязан к пространству и распространяется в вакууме всегда с одной скоростью, а движется источник или стоит - импульс он фотонам не передает.

    Кто знаком с Конвеевской игрой "Жизнь" (или любыми клеточными автоматами), знает, что изменения на поле не могут распространяться быстрее, чем 1 клетка/ход. Как и скорость любого взаимодействия в нашей Вселенной не может превысить c. Что наводит на мысли о жизни внутри симуляции. Когда меня спрашивают, почему нельзя двигаться с бесконечной скоростью, я отвечаю "это защита от оверклокинга".


    1. Wizard_of_light
      30.10.2023 17:26
      +1

      свет привязан к пространству

      Так просто от относительности не отвязаться. Представьте, что два наблюдателя с одинаковыми фонариками движутся и наблюдают друг за другом. Каждый из них видит свой фонарик нормальным, а фонарик другого -искажённым. Нам в итоге придётся предположить, что у каждого из них своё пространство, к которому "привязан свет".


      1. wmlab
        30.10.2023 17:26

        Не совсем. Мы наблюдаем не чужой фонарик, а излученные им волны. То есть, если мы "неподвижны" относительно пространства, то видим волны неискаженными. А если движемся, то частота будет выше или ниже в зависимости от направления нашего движения. Это Доплер, а не Эйнштейн.


        1. Wizard_of_light
          30.10.2023 17:26
          +2

          Так в этом и проблема - пространство неразмечено, за него не зацепиться. Мы можем узнать только свою скорость относительно источника излучения, и то если нам известна частота излучения в покое. Если мы зафиксировали только монохроматическое или чернотельное излучение, мы не можем даже сказать, есть ли у него доплеровский сдвиг.


          1. ksbes
            30.10.2023 17:26

            Если мы зафиксировали только монохроматическое или чернотельное излучение, мы не можем даже сказать, есть ли у него доплеровский сдвиг.

            А как по вашему мы фиксируем расширение вселенной? Именно по доплеровским сдвигам монохроматического (линии испускания/поглощения) и чернотельного излучений!


            1. Wizard_of_light
              30.10.2023 17:26
              +1

              А вот так - у нас есть линии испускания/поглощения элементов, и мы знаем их частоты для неподвижного источника. Если мы зафиксировали излучение со сплошным спектром без характерных линий, мы можем только температуру излучения определить. А имеет ли такую температуру источник, или она у нас из-за доплеровского сдвига изменилась, и насколько - не можем сказать.


              1. ksbes
                30.10.2023 17:26
                -1

                Ну вообще-то экспоненциальный закон излучения чёрного тела не обладает симметрией по отношению к операции сдвига и/или сжатия. Т.е. два сигнала с одинаковыми максимумами - один от неподвижного чёрного тела, а другой от движущегося будут отличаться по форме. У более горячего тела пик по длинам волн более выражен (или, что тоже самое, более толстый "хвост" по частотам)


                1. Wizard_of_light
                  30.10.2023 17:26
                  +2

                  На первый взгляд так кажется, но внезапно, нет. Спектр черного тела релятивистским доплеровским сдвигом преобразуется в такой же спектр черного тела с другой температурой.


                  1. MishaRash
                    30.10.2023 17:26

                    Спектр черного тела релятивистским доплеровским сдвигом преобразуется в такой же спектр черного тела с другой температурой.

                    Это правда.

                    Красное смещение рекомбинации мы бы не смогли узнать из одних измерений температуры реликтового излучения сегодня, оно получается из подробного физического моделирования.


              1. MishaRash
                30.10.2023 17:26
                +1

                А вот так - у нас есть линии испускания/поглощения элементов, и мы знаем их частоты для неподвижного источника.

                Кроме того, линий нужно как минимум две, потому что одна отдельная может оказаться практически чем угодно в системе покоя источника, а вот соотношения частот разных (особенно близких) линий уже достаточно характерны.


          1. wmlab
            30.10.2023 17:26

            Да, пространство не размечено, но скорость относительно него все-таки измеряют. По определению, реликтовое излучение идет равномерно со всех сторон. Если оно с одной стороны слегка синее, а с другой - слегка красное, можно вычислить и скорость и даже направление. Так и был обнаружен Великий Аттрактор.


  1. antiquar
    30.10.2023 17:26
    +2

    По этому вопросу маст рид статья Окуня в УФН https://www.mathnet.ru/php/archive.phtml?wshow=paper&jrnid=ufn&paperid=7675&option_lang=rus


  1. wifage
    30.10.2023 17:26
    -4

    Какая то куча-мала.. Каким образом энергия объекта влияет на его скорость? Причем здесь постоянная скорости света? Все же просто. Если объект бесконечно ускоряется, его скорость бесконечно растет. Возможно, что на него начинают влиять реактивные параметры среды на более высоком уровне. Но это не означает, что скорость любого объекта с постоянным ускорением не стремится к бесконечности.


  1. CBET_TbMbI
    30.10.2023 17:26

    Тут уместрее говорить, не о увеличении массы, а об увеличении гравитационного воздействия на другие предметы. Сам объект, вряд ли почувствует, что стал массивнее.


  1. Zara6502
    30.10.2023 17:26

    • А можно выстрелить фотонами прямо противоположно направлению движения, и всё равно они будут двигаться со скоростью 299 792 458 м/с.

    но поменяется длина волны, что в целом тоже интересно


  1. AlekseiMorozov19730316Ru
    30.10.2023 17:26
    -3

    Независимо от того, кто вы, где вы находитесь и как быстро вы движетесь, законы физики будут выглядеть для вас точно так же, как и для любого другого наблюдателя во Вселенной.

    :) Не совсем точно. Пространство-время-эфир (то есть пространство-время-эфироны) и его закономерности абсолютны. Закономерности же полевого взаимодействия объектов послеэфиронных уровней материи являются в некотором смысле и в некоторой мере "относительными". Это следует из концепции "статических цепных заторных мультиэфиронных коллизий в абсолютном пространстве с относительно-симметричным вещественно-полевым паттерном быстродействующего последовательного перераспределения общей инерции между коллизионными эфиронами", являющейся основой "Эфирной Теории Относительности" из "Эфирной Теории Всего".

    Заманчиво, но в конечном счёте неверно, сваливать несоответствие между классическим и релятивистским миром на идею релятивистского изменения массы.

    :) Ну, а здесь уже явное непонимание наблюдаемой реальности. "Мир" (то есть пространство-время-эфир) - он один - "классический", то есть "абсолютный". В увеличении же "массы эфирного гиперобъекта" (то есть "количества эфиронов в эфирном гиперобъекте") при увеличении его скорости в абсолютном пространстве нет ничего удивительного. С ростом абсолютной скорости эфирного гиперобъекта растёт и частота мультиэфиронных коллизий эфиронов эфирного гиперобъекта с внешними эфиронами, то есть растёт масса эфиронных заторов эфирного гиперобъекта, что и приводит к росту массы эфирного гиперобъекта и его эфирного поля.


    1. PrinceKorwin
      30.10.2023 17:26
      +4

      статических цепных заторных мультиэфиронных коллизий в абсолютном пространстве с относительно-симметричным вещественно-полевым паттерном быстродействующего последовательного перераспределения общей инерции между коллизионными эфиронами", являющейся основой "Эфирной Теории Относительности" из "Эфирной Теории Всего"

      Скажите. А вы мастер классы не проводите? Тоже хочу научиться гнать лютую дичь с умным видом :)


    1. Yuuri
      30.10.2023 17:26

      Высочайшие достижения нейтронной мегалоплазмы! Ротор поля наподобие дивергенции градуирует себя вдоль спина и там, внутре, обращает материю вопроса в спиритуальные электрические вихри, из коих и возникает синекдоза гравитации!


      1. AlekseiMorozov19730316Ru
        30.10.2023 17:26
        -1

        :) Эфирная Теория Всего - это самоочевидная современная классическая эфирная теория материи и сознания.
        Напомню Эфирную Теорию Всего для первого уровня материи в упрощенных односложных формулировках.

        1. Материя существует в пространстве и во времени.

        2. Пространство, время, материя - абсолютны.

        3. Материя состоит из первичных частиц.

        4. Эфироны - это первичные частицы материи.

        5. Эфир - это эфироны.

        6. Материя и эфир - это одно и то же.

        7. Эфирон всегда занимает некоторый объём пространства.

        8. Эфироны не могут одновременно занимать одну и ту же область пространства.

        9. Эфирон может перемещаться в пространстве равномерно и прямолинейно.

        10. Столкновение эфиронов - это попытка занять одну и ту же область пространства несколькими эфиронами.

        11. При столкновении эфироны всегда останавливаются в абсолютном пространстве.

        12. Cтолкновение эфиронов длится некоторое время.

        13. "Столкновение эфиронов" и "эфиронная коллизия" - это одно и то же.

        14. "Столкнувшийся эфирон" и "коллизионный эфирон" - это одно и то же.

        15. Свободный эфирон - это эфирон до столкновения и после столкновения.

        16. "Скорость свободного эфирона" равна "инерции свободного эфирона".

        17. Эфиронный затор - это столкнувшиеся эфироны.

        18. Скорость коллизионного эфирона в абсолютном пространстве всегда равна нулю.

        19. Эфиронный затор всегда покоится в абсолютном пространстве.

        20. Свободный эфирон может столкнуться с эфиронным затором.

        21. Столкнувшийся с эфиронным затором эфирон присоединяется к эфиронному затору.

        22. Во время столкновения эфиронов происходит перераспределение их общей инерции.

        23. "Перераспределение общей инерции коллизионных эфиронов" и "разрешение эфиронной коллизии" - это одно и то же.

        24. В эфиронном заторе перемещение инерции между двумя эфиронами происходит попеременно (сначала от первого ко второму, затем от второго к первому) со скоростью быстродействия пространства-времени-эфира.

        25. Продолжительность столкновения эфиронов зависит от количества столкнувшихся эфиронов и пространственно-временного паттерна столкновения.

        26. Общая инерция коллизионных эфиронов в эфиронном заторе сохраняется.

        27. Общая скорость свободных эфиронов до их столкновения равна общей скорости свободных эфиронов после их столкновения.

        28. Перераспределение общей инерции коллизионных эфиронов имеет радиальный относительно-симметричный паттерн.

        29. Чем дальше эфирон от центральных эфиронов эфиронного затора, тем выше его скорость относительно центральных эфиронов после столкновения.

        30. Эфиронная коллизия завершается после перераспределения общей инерции коллизионных эфиронов.

        31. После эфиронной коллизии эфирон становится свободным и может начать перемещаться в абсолютном пространстве.

        Уровни материи согласно Эфирной Теории Всего.

        1. Эфиронный уровень материи. Эфиронные заторы.

        2. Групповой уровень материи. Эфирное поле.

        3. Спиральный уровень материи. Электрическое поле.

        4. Мажорный уровень материи. Заряд. Магнитное поле. Трёхуровневое эфирное излучение.

        5. Гиператомный уровень материи. Гиператомная иерархичность.

        6. Молекулярный уровень материи. Молекулярная квазииерархичность. Молекулярная проторепликация. Молекулярная репликация.

        7. Телесный уровень материи. Телесная квазииерархичность. Нейронная эфирная ассоциативность.

        8. Небесный уровень материи. Небесная иерархичность.


        1. ksbes
          30.10.2023 17:26

          Как-то опасненько в такой вселенной жить. Есть ненулевая вероятность, что все эфирионы слипнутся в одну гипер-коллизию, которая будет длиться вечно. Да и вообще - эфирионы в виде коллизий и будут существовать большую часть времени. Т.е. основная часть эфира будет в "безмассовом" состоянии.


  1. elprog74
    30.10.2023 17:26
    +1

    тот факт, что свет является электромагнитной волной, означает:

    что при распространении в пространстве она создаёт электрические и магнитные поля, эти поля колеблются, синфазно и под углом 90° друг к другу

    А как при этом ведёт себя плотность потока энергии(она же вектор умова-пойнтинга)?

    Это вектор периодически получается тождественно равным нулю.? :))


    1. ksbes
      30.10.2023 17:26
      +1

      И в чём проблема? Поле же не статично - эти "нулевые узлы" движутся. Т.е. энергия как и положено в волне то ускоряется то останавливается - с волнами на поверхности воды тоже самое происходит (там кинетическая энергия воды может и обратно быть направленна в определённые моменты фазы).
      А при смене координат вектор Пойнтинга сохраняться не обязан (так же как и кинетическая энергия воды в волнах)


      1. elprog74
        30.10.2023 17:26
        +1

        И в чём проблема?

        т.е. передаваемая энергия сейчас есть, а через такт - нету, а потом снова есть, и снова нету? А где же ЗСЭ (закон сохранения энергии)? Отошел покурить? некогда ему за каждой волной следить? :)


        1. ksbes
          30.10.2023 17:26

          Энергия не обязана сохранятся в точке. Она сохраняется в волне целиком - перераспределяется между различными её частями. А если так рассуждать, как вы, то у вас и ветряк энергию из ниоткуда берёт!


          1. DrSmile
            30.10.2023 17:26
            +2

            Энергия сохраняется именно в точке (если забыть про энергию гравитационных волн в ОТО): поток энергии из бесконечно малого объема определяет изменение плотности энергии в этом объеме. А в прерывистом векторе Пойнтинга нет ничего удивительного: плотность энергии там тоже прерывистая. В минимуме нет ни энергии, ни ее потока — все абсолютно логично. Можно считать круговую поляризацию более фундаментальной — там энергия и ее поток непрерывны.


            1. elprog74
              30.10.2023 17:26

              Можно считать круговую поляризацию более фундаментальной — там энергия и ее поток непрерывны.

              Иными словами нам впаривают в учебнике кривую картинку? Причём не первый год.

              Или в случае линейной поляризации что-то радикально меняется?


              1. MishaRash
                30.10.2023 17:26
                +2

                Иными словами нам впаривают в учебнике кривую картинку? Причём не первый год.

                В чём кривизна картинки, в предпочтении линейной поляризации?

                С ней плотность энергии и вектор Пойнтинга, строго говоря, не прерывисты, а изменяются плавно, как квадрат синуса/косинуса, но обычно очень быстро (с удвоенной частотой волны). При этом дифференциальная форма закона сохранения энергии в вакууме

                \frac{\partial U}{\partial t} + \nabla \cdot \vec S = 0

                выполняется.


                1. elprog74
                  30.10.2023 17:26

                  В чём кривизна картинки, в предпочтении линейной поляризации

                  Кривизна в том, что при круговой поляризации плотность потока энергии непрерывна, а при линейной, вдруг, с какого то перепуга, становится пульсирующей. И нет ответа на вопрос куда девается энергия при нуле и откуда она берётся в максимальных значениях.

                  Я, конечно, знаю, что в "военное время значение пи может достигать четырёх" (С), а ЗСЭ может отойти в сторонку и покурить. Но как то остаётся ощущение неправильности используемой модели.. :)


                  1. MishaRash
                    30.10.2023 17:26
                    +1

                    Похоже, что вам плохо пояснили именно закон сохранения энергии. Она не обязана быть постоянной в каком-то фиксированном объёме, если только этот участок совершенно не изолирован от всего остального. А энергия в принципе движется/течёт, вектор Умова-Пойнтинга — это результирующий поток электромагнитной энергии. Закон сохранения некой величины в том, что скорость её изменения в неком объёме равна скорости втекания минус скорости вытекания; результирующая последних двух даётся потоком вектора потока величины (извиняюсь за тавтологию, но такие уж термины) через поверхность-границу того объёма.

                    Кроме того, пульсация получается тогда, когда вы смотрите на область, фиксированную в координатах в некоторой допустимой в СТО системе отсчёта. Если вы будете следить за фиксированным фрагментом волны, который движется со скоростью света, фаза останется той же, а вместе с ней и плотность энергии, и вектор Умова-Пойнтинга. Из такого движущегося вместе с волной объёма энергия не вытекает, и в него не втекает.

                    То есть на вопрос "почему изменилась плотность энергии электромагнитной волны в текущей точке?" есть два правильных ответа с разных точек зрения:

                    • в точку притекло больше/меньше энергии, чем утекло (причём в плоской волне притекает сзади, а утекает вперёд);

                    • прошлый фрагмент волны сдвинулся вперёд, а сзади пришёл другой.

                    Вообще рассмотрение фиксированного объёма в координатах называется эйлеровским подходом (в гидродинамике), а "слежка" за фиксированным "куском" движущегося вещества — лагранжевым.


                    1. MishaRash
                      30.10.2023 17:26

                      Кроме того, можно вспомнить, что линейно поляризованная синусоидальная электромагнитная волна излучается, например, гармонически колеблющимся зарядом (двигающимся туда-сюда по одной линии), а мощность излучения (как целиком, так и в каком-то выделенном направлении) тогда пропорциональна квадрату ускорения. Ускорение при колебаниях тоже меняется синусоидально в противофазе с положением, вот и получается, что такой заряд излучает с переменной мощностью, периодически падающей до нуля, и по мере того как информация о той или иной фазе доходит до определённой точки вместе с электромагнитной волной, плотность энергии и поток энергии там меняются соответственно.

                      А круговая поляризация создаётся, например, зарядом, движущимся по кругу перпендикулярно линии зрения, при этом (перпендикулярное линии зрения) ускорение постоянно и мощность тоже.


                      1. elprog74
                        30.10.2023 17:26

                        линейно поляризованная синусоидальная электромагнитная волна излучается, например, гармонически колеблющимся зарядом (двигающимся туда-сюда по одной линии), а мощность излучения (как целиком, так и в каком-то выделенном направлении) тогда пропорциональна квадрату ускорения. Ускорение при колебаниях тоже меняется синусоидально в противофазе с положением, вот и получается, что такой заряд излучает с переменной мощностью, периодически падающей до нуля,

                        Спасибо. Это понятно.

                        Но тогда возникает трабл с круговой поляризацией, - к примеру спиральная антенна - тот же кусок провода скрученный в спираль и в нем тот же самый гармонически колеблющийся заряд. И почему же говорится(чуть выше), что при круговой поляризации

                        ускорение постоянно и мощность тоже.

                        ?


          1. elprog74
            30.10.2023 17:26
            +1

            Энергия не обязана сохранятся в точке. Она сохраняется в волне целиком - перераспределяется между различными её частями.

            Умова Пойнтинга ничего про это не знает. В этот момент энергия есть, в следующий нет. Куда она делась?


  1. AdiMaste
    30.10.2023 17:26
    +1

    Все пишут одно и то же...

    Автор, поразмышляй лучше на следующую тему: все системы отсчёта равнозначны и это не корабль удаляется от земли, а земля от корабля. И в то время когда для наблюдателя с Земли замедляется время на корабле, для наблюдателя с корабля замедляется время на Земле. На земле пройдёт тысяча лет, в то время когда на корабле всего год, но и на корабле пройдёт тысяча лет, в то время когда на земле пройдёт всего год...


    1. Wizard_of_light
      30.10.2023 17:26
      +1

      Да, до тех пор пока они равномерно и прямолинейно относительно друг друга летят. Но как только кто-нибудь из них решит резко поменять скорость (не будем показывать пальцем, но это скорее всего будет корабль, а не Земля), тут-то все эффекты и станут несимметричными. Если мы рассмотрим группу кораблей, то они, хитро маневрируя, могут посетить несколько разных мест и встретится без парадоксов относительно друг друга (но у внешних наблюдателей могут возникнуть разночтения по поводу того, какой корабль когда где был).


      1. AdiMaste
        30.10.2023 17:26
        -1

        Технически, ведь нет разницы между тем ускорится ли ракета или планета. Они будут удаляться друг от друга с одинаковым ускорением.

        Поэтому возникнет ситуация. При полете от одной звезды к другой и обратно и возникнут непонятки. Но скорей всего они вернутся довольно быстро.

        Но вот другая ситуация довольно интересна: движение в воронке. Прямолинейное но по кругу. Возьмём для примера галактику. Звезды возле центра движутся быстрее, у окраин медленнее. Соответственно по ООП, время для тех что ближе к центру будет замедленно. Для себя они движутся с конкретной скоростью. Для наблюдателя по центру - более близкие будут двигаться медленнее чем на самом деле, а у окраин быстрее чем на самом деле.

        Вспоминаем способы подсчёта масс, и понимаем что неизвестная тёмная материя с энергией это лишь фантомное следствие физики. Вопрос только в размерах метра и секунды.


        1. Wizard_of_light
          30.10.2023 17:26
          +1

          А вот есть, ускорение, в отличие от скорости, неинвариантно, мы всегда можем указать тело, на которое действует ускоряющая сила.


          1. MishaRash
            30.10.2023 17:26

            Да, специальная теория относительности работает только в инерциальных системах отсчёта, движущихся с постоянной скоростью друг относительно друга, без ускорения. Иначе нужно применять общую теорию относительности. Достаточно аналогично ньютоновской механике, где ускорения вызывают дополнительные, "фиктивные" силы инерции, пропорциональные массе тела, только тут уже сложнее с искривлениями пространства-времени (которое в СТО плоское).


            1. ksbes
              30.10.2023 17:26

              Это не так. СТО (как собственно и физика Ньютона) прекрасно умеет работать в ускоряющихся системах безо всякой ОТО. ОТО надо подтягивать, только когда ускорения большие или расстояния гигантские, плотности масс-энергии большие, когда расхождения уже будут существенными. А, например, для масштабов 1g, 100 св.лет вполне себе и СТО "отработает" (с неточностями порядка процента - долей процента).
              Только там всякие не очень удобные весёлости вылазят - вроде продольных/поперечных масс и т.д.


              1. MishaRash
                30.10.2023 17:26
                +1

                Тут нужно аккуратно. В парадоксе близнецов можно представить, что корабль сначала очень плавно ускоряется, а потом очень плавно замедляется до остановки, и повторяет процедуру для возвращения. Если для СО корабля СТО "примерно" работает, то в ней по-прежнему получится, что на Земле должно пройти меньше времени, а с точки зрения Земли наоборот.

                Хотя, вероятно, критерий малости ошибок от применения СТО в неинерциальной СО формулируется как a\cdot t\ll c (произведение характерного ускорения на характерное время много меньше скорости света) и в примере выше его придётся нарушить.


                1. ksbes
                  30.10.2023 17:26

                  Ну там по формуле ОТОшного замедления времени получается условие что-то вроде aL << c^2 (L - характерное расстояние между оборзевателями). Его посложнее нарушить.


  1. boorrdenis
    30.10.2023 17:26

    Почему в световых часа, образованных фотоном, отражающимся от двух зеркал (последняя картинка) рассматривают фотон, а не волну. Ведь же рассматривая волну все встанет на свои места. И тогда получится, что стартовавший фотон на стационарной конструкции зеркал не тот же фотон что на подвижной конструкции. Это же очевидно! Нет?


    1. Wizard_of_light
      30.10.2023 17:26
      +1

      Проблема в том, что для внешнего наблюдателя это "не тот фотон", а для наблюдателя, летящего вместе с часами - тот.


  1. vasilijmooduckovic
    30.10.2023 17:26
    -1

    Первое, что важно понять, - это то, что принцип относительности, независимо от того, как быстро вы движетесь или где находитесь, остаётся верным всегда: законы физики действительно одинаковы для всех, независимо от того, где вы находитесь и когда проводите измерения.

    ну хз попробуйте подпрыгнуть на Земле и на Луне например, then tell me about it


  1. ss-nopol
    30.10.2023 17:26

    Если я правильно всё понимаю (поправьте есть что), хотя друг относительно друга сигналы/тела никогда не будут удаляться быстрее скорости света, но если запустить влево и вправо по электрону с околосветовой скоростью (или по фотону), то расстояние между ними относительно наблюдателя будет расти со скоростью выше скорости света. Соответственно если слева и справа к наблюдателю летит по электрону с околосветовой скоростью, то они будут сближаться (и столкнутся) со скоростью выше скорости света в системе отсчёта связанной с наблюдателем. А друг относительно друга при этом они будут двигаться медленнее скорости света.

    А что при этом будет с выделенной энергией? Получается она будет разной в разных системах отсчёта?


  1. porn
    30.10.2023 17:26

    Анимация очень раздражает когда читаешь текст. Хоть под спойлеры спрятать бы.